2. Capital Budgeting Methods

Share Embed Donate


Short Description

CMA...

Description

Part 2 : Capital Budgeting Methods Question 1 - CIA 590 IV.49 - Capital Budgeting Methods A firm's optimal capital structure A. Maximizes the price of the firm's stock. B. Minimizes the firm's tax liability. C. Maximizes the firm's degree of financial leverage. D. Minimizes the firm's risk. A. The capital structure that maximizes the share price is the optimal capital structure. If the share price is at its highest, that means that management has properly balanced the risk and returns in its capital structure and investors value this structure the most. B. The lowest tax liability may not be the best capital structure for the company because the lowest tax rate would be achieved with all debt financing. All debt financing may not provide the lowest cost of capital and it may lead to high levels of risk for the company because of all of the fixed interest payments. C. The maximization of financial leverage would lead to high levels of risk and would therefore probably not be the best capital structure. D. The lowest risk may not be the best capital structure for the company because the lowest tax rate would be achieved with all equity financing. All equity financing may not provide the lowest cost of capital and it may lead to investors not wanting to be owners when the ownership share is so diluted from all of the shares that have been issued.

Question 2 - CIA 597 IV.40 - Capital Budgeting Methods A firm with an 18% cost of capital is considering the following projects (on January 1, year 1): Year 5 January 1, Year 1 December 31, Year 5 Project Internal Cash Inflow Cash Outflow Rate of Return (000's Omitted) (000's Omitted) Project A

$3,500

$7,400

16%

Project B

4,000

9,950

?

Present Value of $1 Due at the End of "N" Periods N

12%

4

.6355 .5921 .5718 .5523 .5158 .4823 .4230

14%

15%

16%

18%

20%

22%

5

.5674 .5194 .4972 .4761 .4371 .4019 .3411

6

.5066 .4556 .4323 .4104 .3704 .3349 .2751

Using the net-present-value (NPV) method, project A's net present value is A. $(265,460) B. $23,140 C. $316,920 D. $(316,920) A. There is only one cash inflow to this project, and it occurs 5 years after the initial cash outflow. The firm's cost of capital is 18%. Therefore, the correct Present Value of $1 factor (from the table given) to use in discounting the cash inflow is .4371. The present value of the cash inflow is .4371 × $7,400,000, or $3,234,540. Subtracting the initial investment of $3,500,000 from the present value of the cash inflow of $3,234,540, we get $(265,460). (c) HOCK international, page 1

Part 2 : Capital Budgeting Methods B. An answer of $23,140 results from discounting the cash inflow at the rate of 16%, which is the project's Internal Rate of Return but which is not the appropriate discount rate to use. C. An answer of $316,920 results from discounting the cash inflow for four years. However, the receipt of the cash occurs five years after the initial investment. D. An answer of $(316,920) results from two errors. One, the factor for four years was used to discount the cash inflow, which was not correct. And two, the present value of the cash inflow was subtracted from the initial cash outflow. Instead, the initial cash outflow should be subtracted from the present value of the cash inflow.

Question 3 - CIA 597 IV.41 - Capital Budgeting Methods A firm with an 18% cost of capital is considering the following projects (on January 1, year 1): Year 5 January 1, Year 1 December 31, Year 5 Project Internal Cash Inflow Cash Outflow Rate of Return (000's Omitted) (000's Omitted) Project A

$3,500

$7,400

16%

Project B

4,000

9,950

?

Present Value of $1 Due at the End of "N" Periods N

12%

14%

15%

16%

18%

20%

22%

4

.6355 .5921 .5718 .5523 .5158 .4823 .4230

5

.5674 .5194 .4972 .4761 .4371 .4019 .3411

6

.5066 .4556 .4323 .4104 .3704 .3349 .2751

Project B's internal rate of return is closest to A. 18% B. 16% C. 15% D. 20% A. 18% is the company's cost of capital, which is given in the question. B. 16% is approximately the internal rate of return for project A, but the question asks for the internal rate of return for project B. C. The internal rate of return is the discount rate at which the present value of the expected cash inflows from a project equals the present value of the expected cash outflows, or the discount rate at which the net present value is zero. A positive NPV would result from using a discount rate of 15%, so that cannot be the project's IRR. D. The internal rate of return is the discount rate at which the present value of the expected cash inflows from a project equals the present value of the expected cash outflows, or the discount rate at which the net present value is zero. To determine the internal rate of return from the information given, we need to first know what discount factor for five years would result in a present value of $9,950 that is equal to $4,000. To arrive at that factor, we divide $4,000 by $9,950, and we get .402. We then look along the line of factors for five years on the (c) HOCK international, page 2

Part 2 : Capital Budgeting Methods factor table given to locate a factor close to .402. That is .4019, which is in the 20% column. Thus, the internal rate of return is closest to 20%.

Question 4 - CMA 1278 5.12 - Capital Budgeting Methods Future, Inc. is in the enviable situation of having unlimited capital funds. The best decision rule, in an economic sense, for it to follow would be to invest in all projects in which the A. Net present value is greater than zero. B. Payback reciprocal is greater than the internal rate of return. C. Accounting rate of return is greater than the earnings as a percent of sales. D. Internal rate of return is greater than zero. A. If a company has unlimited capital funds, it should invest in all projects in which the net present value is greater than zero, assuming none of the projects are mutually exclusive. B. The Payback Reciprocal is 1 divided by the payback period. It can be used to give a very rough indication of an internal rate of return, if the cash flows from the project are equal in every period and if the project life is at least twice as long as the payback period. However, comparing the payback reciprocal to the internal rate of return is a meaningless comparison. C. Earnings as a percent of sales is meaningless in a capital budgeting analysis. So comparing an accounting rate of return to it is also meaningless. D. If a project's internal rate of return is greater than zero but lower than the company's required rate of return, the project should not be undertaken.

Question 5 - CMA 1278 5.8 - Capital Budgeting Methods Carco, Inc. wants to use discounted cash flow techniques when analyzing its capital investment projects. The company is aware of the uncertainty involved in estimating future cash flows. A simple method some companies employ to adjust for the uncertainty inherent in their estimates is to A. Prepare a direct analysis of the probability of outcomes. B. Use accelerated depreciation. C. Adjust the minimum desired rate of return. D. Increase the estimates of the cash flows. A. Although the preparation of a direct analysis of the probability of outcomes may be used to address uncertainty, it is not a simple method, and this question asks for a simple method. B. The use of accelerated depreciation is not a technique used to address uncertainty in capital budgeting. C. A company adjusts for the uncertainty inherent in its estimates by increasing the required rate of return used to discount the future expected cash flows. A higher discount rate will require higher expected future cash flows in order for the investment to be acceptable. As a result, fewer investments will be acceptable. D. Increasing estimates of future expected cash flows arbitrarily is not an acceptable method of addressing uncertainty in the capital budgeting process.

(c) HOCK international, page 3

Part 2 : Capital Budgeting Methods

Question 6 - CMA 1286 5.4 - Capital Budgeting Methods A manager wants to know the effect of a possible change in cash flows on the net present value of a project. The technique used for this purpose is A. Cost behavior analysis. B. Sensitivity analysis. C. Risk analysis. D. Return on investment analysis. A. Cost behavior analysis is not used to determine the effect of a possible change in cash flows on the net present value of a project. B. Sensitivity analysis is used to determine how an amount will change if factors that were involved in predicting that amount change. C. Risk analysis is not used to determine the effect of a possible change in cash flows on the net present value of a project. D. Return on Investment (ROI) is used to analyze the profitability of a company or one of its segments, not for analyzing the effect of a possible change in cash flows on the net present value of a project.

Question 7 - CMA 1290 4.13 - Capital Budgeting Methods The technique that recognizes the time value of money by discounting the after-tax cash flows for a project over its life to time period zero using the company's minimum desired rate of return is called the A. Payback method. B. Net present value method. C. Accounting rate of return method. D. Average rate of return method. A. The payback method explicitly does not recognize the time value of money. B. Net present value of a project is calculated by discounting the after-tax expected cash flows for the project over its life to time period zero using the company's minimum required rate of return. The present value of the future expected cash inflows minus the net initial investment equals the net present value. C. The accounting rate of return is the ratio of the amount of increased book income to the required investment. Since this method uses accrual accounting income, it includes depreciation. However, it does not take into account the time value of money. D. The average rate of return is not a technique that recognizes the time value of money by discounting the after-tax cash flows for a project.

Question 8 - CMA 1290 4.14 - Capital Budgeting Methods The technique that reflects the time value of money and is calculated by dividing the present value of the future net after-tax cash inflows that have been discounted at the desired cost of capital by the initial cash outlay for the investment is called the

(c) HOCK international, page 4

Part 2 : Capital Budgeting Methods A. Profitability index method. B. Capital rationing method. C. Accounting rate of return method. D. Average rate of return method. A. The profitability index is a benefit-cost ratio. It is the ratio of the present value of net future expected cash flows, discounted at the required rate of return, to the amount of the initial investment. B. Capital rationing occurs when the amount of capital funds available to invest is limited. C. The accounting rate of return is the ratio of the amount of increased book income to the required investment. Since this method uses accrual accounting income, it includes depreciation, and it also does not take into account the time value of money. D. The average rate of return method is not a technique that reflects the time value of money.

Question 9 - CMA 1290 4.15 - Capital Budgeting Methods The technique that measures the estimated performance of a capital investment by dividing the project's annual after-tax net income by the average investment cost is called the A. Internal rate of return method. B. Accounting rate of return method. C. Capital asset pricing model. D. Average rate of return method. A. The internal rate of return is the discount rate at which the net present value of a capital budgeting project is zero. It does not measure the estimated performance of a capital investment by dividing the project's annual after-tax net income by the average investment cost, however. B. The accounting rate of return is the ratio of the amount of increased book income to the required investment. Sometimes the average investment figure is used rather than the total initial investment. This is usually calculated as the initial investment divided by 2. The initial investment is divided by 2 because the investment will have a book value of 0 at the end of the project, and dividing the initial investment by 2 approximates an average of the amount invested over the life of the project. C. The capital asset pricing model may be used to calculate a company's cost of capital, but it is not used to analyze an individual project's performance. D. The average rate of return is not a technique that measures the estimated performance of a capital investment by dividing the project's annual after-tax net income by the average investment cost.

Question 10 - CMA 1290 4.16 - Capital Budgeting Methods The technique that incorporates the time value of money by determining the compound interest rate of an investment such that the present value of the after-tax cash inflows over the life of the investment is equal to the initial investment is called the A. Accounting rate of return method. B. Internal rate of return method. C. Capital asset pricing model.

(c) HOCK international, page 5

Part 2 : Capital Budgeting Methods D. Profitability index method. A. The accounting rate of return is the ratio of the amount of increased book income to the required investment. It does not incorporate the time value of money. B. The internal rate of return is the discount rate at which the net present value of a project is zero. Therefore, it is also the discount rate at which the present value of the after-tax cash inflows over the life of the investment equal the initial investment, assuming that the future expected cash flows are all positive. C. The capital asset pricing model may be used to calculate a company's cost of capital. It is not a technique that incorporates the time value of money by determining the compound interest rate of an investment such that the present value of the after-tax cash inflows over the life of the investment is equal to the initial investment. D. The profitability index is a benefit-cost ratio. It is the ratio of the present value of net future expected cash flows, discounted at the required rate of return, to the amount of the initial investment. It is not an interest rate.

Question 11 - CMA 1290 4.17 - Capital Budgeting Methods The technique that measures the number of years required for the after-tax cash flows to recover the initial investment in a project is called the A. Payback method. B. Net present value method. C. Profitability index method. D. Accounting rate of return method. A. The Payback Method is used to determine the number of periods that must pass before the net after-tax cash inflows from the investment will equal (or "pay back") the initial investment cost. If the expected cash inflows are constant over the life of the project, the payback period is the net initial investment divided by the periodic expected cash flow. If the expected cash inflows are not constant over the life of the project, the cash inflows are added to determine on a cumulative basis when the inflows will equal the outflows. The payback method ignores all cash flows beyond the payback period, does not include the time value of money, and does not include any factor for the cost of capital. However, it is widely used because it is simple and it can be useful when a project is judged to be very risky with uncertain cash flows in the later years. In this case, it may be used to determine how quickly the investment will be recouped so that if necessary, the company can abandon the project without too great a loss. B. The net present value method does not measure the number of years required for the after-tax cash flows to recover the initial investment in a project. The net present value method is a discounted cash flow method which calculates the value of a project by discounting the after-tax expected cash flows for the project over its life to time period zero using the company's minimum required rate of return. The present value of the future expected cash inflows minus the net initial investment equals the net present value. C. The profitability index does not measure the number of years required for the after-tax cash flows to recover the initial investment in a project. The profitability index is a ratio of the present value of the net future cash flows to the amount of the initial investment. If a project has a positive net present value, the profitability index will be above 1.00. If it has a negative net present value, it will have a profitability index of less than 1.00. D. The accounting rate of return does not measure the number of years required for the after-tax cash flows to recover the initial investment in a project. The accounting rate of return is a ratio of the increase in expected annual average after tax accounting net income to the net initial investment. This method uses accrual accounting income, so depreciation is included in the expenses. In addition, it does not take into account the time value of money.

(c) HOCK international, page 6

Part 2 : Capital Budgeting Methods

Question 12 - CMA 1291 4.1 - Capital Budgeting Methods Yipann Corporation is reviewing an investment proposal. The initial cost as well as other related data for each year are presented in the schedule below. All cash flows are assumed to take place at the end of the year. The salvage value of the investment at the end of each year is equal to its net book value, and there will be no salvage value at the end of the investment's life. Investment Proposal Initial Cost and Year Book Value

Annual Net After-Tax Cash Flows $

Annual Net Income

0

$105,000

1

70,000

50,000

0

$

15,000

0

2

42,000

45,000

17,000

3

21,000

40,000

19,000

4

7,000

35,000

21,000

5

0

30,000

23,000

Yipann uses a 24% after-tax target rate of return for new investment proposals. The discount figures for a 24% rate of return are given. Present Value of Present Value of Annuity of $1 $1 Received at Received at End Year the End of Period of Each Period 1

.81

.81

2

.65

1.46

3

.52

1.98

4

.42

2.40

5

.34

2.74

6

.28

3.02

7

.22

3.24

The traditional payback period for the investment proposal is A. .875 years. B. 1.833 years. C. Over 5 years. D. 2.250 years. A. An answer of .875 results from combining the book values and the net after-tax cash flows instead of using the after tax cash flows alone for the years subsequent to year 0. B. An answer of 1.833 years results from using the yearly book values instead of the after-tax cash flows for the years subsequent to year 0. C. An answer of "over 5 years" results from using the annual net income amounts instead of the after-tax cash flows for the years subsequent to year 0.

(c) HOCK international, page 7

Part 2 : Capital Budgeting Methods D. The cash flow analysis is set up as follows:

Year 0 Year 1 Year 2 Initial Investment in Equipment (105,000) After-Tax Cash Flow 50,000 45,000 ------------ ----------------Total After-Tax Cash Flows (105,000) 50,000 45,000 Cumulative Cash Flow (105,000) (55,000) (10,000)

Year 3 Year 4 Year 5 40,000 --------40,000 30,000

35,000 --------35,000 65,000

30,000 --------30,000 95,000

The cumulative cash flow from the project becomes positive during Year 3. Assuming that the cash flows occur evenly throughout the year, the payback period is 2.25 years, calculated as follows: Number of the project year in the final year when cash flow is negative: 2 Plus: a fraction consisting of Numerator = the positive value of the negative cumulative inflow amount from the final negative year, which is 10,000 Denominator = cash flow for the following year, which is 40,000 or: 2 + 10,000/40,000 = 2.25 Note that the present value factors given are irrelevant to answering this question, because the payback method is not a discounted cash flow technique.

Question 13 - CMA 1291 4.10 - Capital Budgeting Methods Crown Corporation has agreed to sell some used computer equipment to Bob Parsons, one of the company's employees, for $5,000. Crown and Parsons have been discussing alternative financing arrangements for the sale. Crown Corporation has offered to accept a $1,000 down payment and set up a note receivable for Bob Parsons that calls for a $1,000 payment at the end of each of the next 4 years. If Crown uses a 6% discount rate, the present value of the note receivable would be A. $2,940 B. $4,465 C. $3,465 D. $4,212 A. An answer of $2,940 results from discounting the entire amount of the note ($4,000) at 8% for four years. This is incorrect for two reasons: (1) The full principle of the note ($4,000) is assumed to be paid at the maturity date in four years. However, the note calls for annual principle payments of $1,000. (2) The discount rate used is 8%. However, the discount rate to be used is 6%. B. An answer of $4,465 results from discounting the note receivable using the present value of an annuity of $1,000 and a 6% rate for 4 years and then adding the $1,000 down payment at its undiscounted value of $1,000. However, the question asks only for the present value of the note receivable, not the present value of the entire transaction. (c) HOCK international, page 8

Part 2 : Capital Budgeting Methods

C. The note calls for four annual payments of $1,000. This is an ordinary annuity, since the payments are due at the end of each period. Therefore, the factor in the present value of an annuity table can be used as it is given, without adjustment. The present value of a four-year ordinary annuity of $1,000, discounted at 6%, is $1,000 × 3.465, or $3,465. D. An answer of $4,212 results from using the present value of an annuity factor for 6% for 5 years. However, the note is for four years.

Question 14 - CMA 1291 4.11 - Capital Budgeting Methods Crown Corporation has agreed to sell some used computer equipment to Bob Parsons, one of the company's employees, for $5,000. Crown and Parsons have been discussing alternative financing arrangements for the sale. Crown has offered to accept a $1,000 down payment and to set up a note receivable for Bob Parsons that calls for a $1,000 down payment at the end of this year and the next three years. Bob Parsons has agreed to the immediate down payment of $1,000 but would like the note for $4,000 to be payable in full at the end of the fourth year. Because of the increased risk associated with the terms of this note, Crown Corporation would apply an 8% discount rate. The present value of this note would be A. $2,577 B. $2,940 C. $3,940 D. $3,312 A. An answer of $2,577 results from discounting a series of payments of $1,000 at 8% for three years. However, the term of the note is four years, and the note is payable at the end of the term, not in annual payments. B. Using the Present Value of $1 table, the present value of a single $4,000 payment in 4 years is $4,000 × .735, which equals $2,940. C. An answer of $3,940 results from discounting the principal repayment of $4,000 at 8% for four years and adding the down payment of $1,000 without discounting it. However, the question asks for the present value of the note, not the present value of the whole transaction. D. An answer of $3,312 results from discounting a series of payments of $1,000 at 8% for four years. However, the note is payable at the end of the term, not in annual payments.

Question 15 - CMA 1291 4.2 - Capital Budgeting Methods Yipann Corporation is reviewing an investment proposal. The initial cost as well as other related data for each year are presented in the schedule below. All cash flows are assumed to take place at the end of the year. The salvage value of the investment at the end of each year is equal to its net book value, and there will be no salvage value at the end of the investment's life. Investment Proposal Annual Net After-Tax Annual Initial Cost Year and Book Value Cash Flows Net Income 0

$105,000

$

0

$

0 (c) HOCK international, page 9

Part 2 : Capital Budgeting Methods 1

70,000

50,000

15,000

2

42,000

45,000

17,000

3

21,000

40,000

19,000

4

7,000

35,000

21,000

5

0

30,000

23,000

Yipann uses a 24% after-tax target rate of return for new investment proposals. The discount figures for a 24% rate of return are given. Present Value of Annuity of $1 Present Value of $1 Received at Received at End Year the End of Period of Each Period 1

.81

.81

2

.65

1.46

3

.52

1.98

4

.42

2.40

5

.34

2.74

6

.28

3.02

7

.22

3.24

The average annual cash inflow at which Yipann would be indifferent to the investment (rounded to the nearest dollar) is A. $38,321. B. $40,000. C. $21,000. D. $46,667. A. The question is asking for an average annual after-tax cash flow amount that will result in a net present value of zero for the project, because that will be the average annual cash flow level at which Yipann will be indifferent to the investment. We need to look at this as a present value of an annuity problem, because since we are looking for an average annual cash flow amount, all the annual cash flow amounts after year 0 will be the same average amount. The annual cash flows given in the problem are irrelevant, because we are looking for the average annual after-tax cash flow amount that will result in an NPV of zero, given the initial investment in Year 0. Since the initial investment is $105,000 and the project's life is 5 years, we need to know what annuity amount will produce a present value of $105,000 when discounted at 24% for 5 years. Recall that the present value of an annuity is the annuity amount × PV of an annuity factor. We don't know the annuity amount, but we do know the PV of an annuity factor and the present value amount of $105,000. The PV of an annuity factor for 5 years at 24% is given in the problem: 2.74. Thus, the formula is: Annuity Amount × 2.74 = $105,000. Therefore, the Annuity Amount = $105,000 ÷ 2.74, which is equal to $38,321. This means that if the annual after-tax cash flows are all the same and they are $38,321, the NPV will be zero and the company will be indifferent to the investment. B. $40,000 is the average of the five net after tax cash flows already given.

(c) HOCK international, page 10

Part 2 : Capital Budgeting Methods C. An answer of $21,000 results from dividing the initial cost of the asset by the number of years of the project's life. D. An answer of $46,667 results from dividing the initial investment amount of $105,000 by the payback period.

Question 16 - CMA 1291 4.3 - Capital Budgeting Methods Yipann Corporation is reviewing an investment proposal. The initial cost as well as other related data for each year are presented in the schedule below. All cash flows are assumed to take place at the end of the year. The salvage value of the investment at the end of each year is equal to its net book value, and there will be no salvage value at the end of the investment's life. Investment Proposal Annual Net After-Tax Annual Initial Cost Year and Book Value Cash Flows Net Income 0

$105,000

$0

$

0

1

70,000

50,000

15,000

2

42,000

45,000

17,000

3

21,000

40,000

19,000

4

7,000

35,000

21,000

5

0

30,000

23,000

Yipann uses a 24% after-tax target rate of return for new investment proposals. The discount figures for a 24% rate of return are given. Present Value of Annuity of $1 Present Value of $1 Received at Received at End Year the End of Period of Each Period 1

.81

.81

2

.65

1.46

3

.52

1.98

4

.42

2.40

5

.34

2.74

6

.28

3.02

7

.22

3.24

The accounting rate of return for the investment proposal over its life using the initial value of the investment is A. 38.1%. B. 28.1%. C. 18.1%. D. 36.2%. A. An answer of 38.1% results from the average of the after-tax cash flows divided by the net initial investment. However, the after-tax cash flows are not used in calculating the accounting rate of return. B. An answer of 28.1% results from averaging the annual after-tax cash flows and then averaging the annual net incomes, then taking the average of the two averages and dividing it by the net initial investment. However, the after-tax (c) HOCK international, page 11

Part 2 : Capital Budgeting Methods cash flows and annual net incomes should not be averaged together. C. The accounting rate of return is the average annual after-tax net income attributable to the project divided by the net initial investment. The average of the five annual net income amounts given is $19,000 ([$15,000 + $17,000 + $19,000 + $21,000 + $23,000] / 5 = $19,000.) $19,000 / $105,000 = .18095 or 18.1%. (Note: sometimes the average of the initial investment over the life of the project is used, calculated as the initial investment divided by 2. However, this question specifies to use the initial value of the investment, not the average investment.) D. An answer of 36.2% results from using the average amount of the investment over the life of the project (the net initial investment divided by 2). Sometimes, the accounting rate of return is calculated using the average amount of the investment over the life of the project. However, this question specifies to use the initial value of the investment, not the average investment.

Question 17 - CMA 1291 4.6 - Capital Budgeting Methods When ranking two mutually exclusive investments with different initial amounts, management should give first priority to the project A. Whose net after-tax flows equal the initial investment. B. That has the greater accounting rate of return. C. That has the greater profitability index. D. That generates cash flows for the longer period of time. A. If the net after-tax flows (discounted, presumably) equal the initial investment, then the net present value of the project will be zero and the investor will be indifferent to the project. B. The investment that has the greater accounting rate of return may not offer the best return on a discounted cash flow basis. C. The Profitability Index enables us to compare (or rank) the benefit/cost ratios of different sized investments, since the Profitability Index expresses profitability on a percentage basis rather than a total dollar amount basis. It is very useful when we must compare multiple investments that are of different investment amounts. D. The investment that generates cash flows for the longer period of time may not offer the best return.

Question 18 - CMA 1291 4.7 - Capital Budgeting Methods The net present value (NPV) method and the internal rate of return (IRR) method are used to analyze capital expenditures. The IRR method, as contrasted with the NPV method, A. Is considered inferior because it fails to calculate compounded interest rates. B. Is preferred in practice because it is able to handle multiple desired hurdle rates, which is impossible with the NPV method. C. Assumes that the rate of return on the reinvestment of the cash proceeds is at the indicated rate of return of the project analyzed rather than at the discount rate used. D. Incorporates the time value of money whereas the NPV method does not. A. The internal rate of return method does utilize compounded interest rates. B. The internal rate of return method does not handle multiple desired hurdle rates. C. The internal rate of return is the discount rate at which the net present value of a project is zero. The internal (c) HOCK international, page 12

Part 2 : Capital Budgeting Methods rate of return method assumes that the cash proceeds of the investment will be reinvested at the internal rate of return, which may not be the case. D. Both the IRR method and the NPV method incorporate the time value of money.

Question 19 - CMA 1291 4.8 - Capital Budgeting Methods Mercken Industries is contemplating four projects, Project P, Project Q, Project R, and Project S. The capital costs and estimated after-tax net cash flows of each project are listed below. Mercken's desired after-tax opportunity cost is 12%, and the company has a capital budget for the year of $450,000. Idle funds cannot be reinvested at greater than 12%. Project P Project Q Project R Project S Initial cost

$200,000 $235,000 $190,000 $210,000

Annual cash flows Year 1

$93,000

$90,000

$45,000

$40,000

Year 2

93,000

85,000

55,000

50,000

Year 3

93,000

75,000

65,000

60,000

Year 4

0

55,000

70,000

65,000

Year 5

0

50,000

75,000

75,000

$23,370

$29,827

$27,333

$(7,854)

18.7%

17.6%

17.2%

10.6%

1.12

1.13

1.14

0.96

Net present value Internal rate of return Excess present value index

During this year, Mercken will choose A. Projects P and Q. B. Projects P, Q, and R. C. Projects Q and R. D. Projects P, Q, R, and S. A. This is a situation where there appears to be a conflict between the NPV results and the IRR results. Projects P and Q have the higher IRRs, whereas Q and R have the higher NPVs. However, the problem also tells us that the cash inflows from the project will be able to be reinvested at a rate no higher than 12%. Thus, the IRRs for these projects are not accurate. Furthermore, Projects P and Q do not have the highest excess present value (profitability) indices. B. If Mercken chooses Projects P, Q and R, it will exceed its capital budget for the year. C. Project S is not an acceptable project, because it has a negative NPV. That leaves Projects P, Q and R. However, if all three projects were selected, Mercken would exceed its capital budget of $450,000. Therefore, only two of the three projects can be selected. This is a situation where there appears to be a conflict between the NPV results and the IRR results. Projects P and Q have the higher IRRs, whereas Q and R have the higher NPVs. Remember that the IRR assumes that all cash inflows from the project will be able to be reinvested at the internal rate of return. However, this problem tells us that the cash inflows from the project will be able to be reinvested at a rate no higher than 12%. Thus, the IRRs for these projects are not accurate. Therefore, the projects with the highest NPVs should be selected, and those are Projects Q and R. This is confirmed by looking at the excess present value (profitability) indices. Note that Projects Q and R have the highest profitability indices. D. First, if all four projects are accepted, Mercken would exceed its capital budget for the year. And second, Project S is completely unacceptable because it has a negative NPV. Therefore, this answer should be rejected.

(c) HOCK international, page 13

Part 2 : Capital Budgeting Methods

Question 20 - CMA 1292 4.11 - Capital Budgeting Methods The bailout payback method A. Equals the recovery period from normal operations. B. Measures the risk if a project is terminated. C. Eliminates the disposal value from the payback calculation. D. Incorporates the time value of money. A. The bailout payback method does not determine the recovery period from normal operations. It determines the recovery period if operations are abnormal. B. The bailout payback method recognizes the possibility that a project may be ended prematurely and the equipment sold. The after-tax salvage value of the equipment at various dates is included in the cash inflows of the project through the same dates. The use of the bailout payback method gives an indication of the result of terminating the project early. C. The bailout payback method does not eliminate the disposal value from the payback calculation, but it incorporates it at various points in the calculation. D. The bailout payback method, like the payback method, does not incorporate the time value of money.

Question 21 - CMA 1292 4.13 - Capital Budgeting Methods A weakness of the internal rate of return (IRR) approach for determining the acceptability of investments is that it A. Does not consider the time value of money. B. Implicitly assumes that the firm is able to reinvest project cash flows at the firm's cost of capital. C. Implicitly assumes that the firm is able to reinvest project cash flows at the project's internal rate of return. D. Is not a straightforward decision criterion. A. The Internal Rate of Return does consider the time value of money. B. If the required rate of return used in an NPV calculation is the firm's cost of capital, the calculation of the NPV assumes that the firm is able to reinvest the project's cash flows at the same rate. However, the IRR is not the firm's cost of capital. The IRR may be higher or lower than the firm's required rate of return. C. The Internal Rate of Return (IRR) is the discount rate at which the Net Present Value (NPV) of a project is zero. As such, it assumes that the cash flows from the project will be reinvested at the same rate. This is a disadvantage, because the cash flows from the project may not be able to be reinvested at the Internal Rate of Return. D. The Internal Rate of Return is a straightforward decision criterion. If a project's IRR is greater than the firm's required rate of return, the project is acceptable.

Question 22 - CMA 1292 4.14 - Capital Budgeting Methods The profitability index approach to investment analysis

(c) HOCK international, page 14

Part 2 : Capital Budgeting Methods A. Always yields the same accept/reject decisions for mutually exclusive projects as the net present value method. B. Fails to consider the timing of project cash flows. C. Considers only the project's contribution to net income and does not consider cash flow effects. D. Always yields the same accept/reject decisions for independent projects as the net present value method. A. If projects are mutually exclusive or are of differing time periods and initial investment amounts, the PI method may result in a different ranking from that of the NPV method. B. The PI does not fail to consider the timing of project cash flows. On the contrary, it uses the present value of the cash inflows discounted using the firm's required rate of return. Therefore, the timing of the cash flows is included. C. The calculation of PI is based upon cash flow, not net income. D. The Profitability Index (PI) is a benefit/cost ratio. It is the present value of the future net cash inflows divided by the initial net cash investment. A ratio of greater than 1 indicates an acceptable project. Therefore, whenever projects are independent (i.e., not mutually exclusive), the PI will yield the same accept/reject decision as the NPV method, because a positive NPV will result in a PI of greater than 1. The PI method is useful for ranking multiple investment opportunities that are of different investment amounts if the projects are not mutually exclusive.

Question 23 - CMA 1292 4.15 - Capital Budgeting Methods The rankings of mutually exclusive investments determined using the internal rate of return method (IRR) and the net present value method (NPV) may be different when A. Multiple projects have unequal lives and the size of the investment for each project is different. B. The lives of the multiple projects are equal and the size of the required investments are equal. C. The required rate of return is higher than the IRR of each project. D. The required rate of return equals the IRR of each project. A. When projects have unequal lives and the sizes of the investments are different, it is possible that NPV and IRR will rank the projects in a different order. B. When the lives of multiple projects are equal and the size the required investments are equal, the IRR method and the NPV method will return the same accept-reject decision. C. If the required rate of return is higher than the IRR of each project, NPV will be negative and both NPV and IRR will return a decision to reject the project. D. If the required rate of return equals the IRR of each project, NPV will be zero and the NPV method and the IRR method will rank the investments the same.

Question 24 - CMA 1292 4.16 - Capital Budgeting Methods When using the net present value method for capital budgeting analysis, the required rate of return is called all of the following except the A. Discount rate. B. Hurdle rate. C. Cost of capital. D. Risk-free rate.

(c) HOCK international, page 15

Part 2 : Capital Budgeting Methods A. The required rate of return is the discount rate used in a capital budgeting analysis. B. The required rate of return may be called the "hurdle rate" because it is the minimum rate of return that is acceptable for an investment. A firm should invest money in a project only if the project provides a higher rate of return than this rate. Investments with a return higher than the hurdle rate will increase the value of the firm and thus stockholders' wealth. C. The required rate of return may be equal to the firm's cost of capital, if the firm has not seen fit to adjust its cost of capital to reflect higher or lower risk. D. The required rate of return, which is the rate used to discount future cash flows in a capital budgeting analysis, is not the risk-free rate. There is risk inherent in all capital budgeting projects, and the required rate of return incorporates a risk premium.

Question 25 - CMA 1292 4.19 - Capital Budgeting Methods The proper discount rate to use in calculating certainty equivalent net present value is the A. Cost of capital. B. Risk-adjusted discount rate. C. Risk-free rate. D. Cost of equity capital. A. The firm's cost of capital is not used in calculating certainty equivalent NPV. B. A risk-adjusted discount rate is not used in calculating certainty equivalent NPV. C. The goal of the certainty equivalent approach is to find the smallest cash flow in each period that would be acceptable in place of that period's risky cash flow, if that smaller cash flow can be depended upon absolutely. The certainty equivalent approach adjusts risky after-tax cash flows to a level judged by the decision-maker to be certain of attainment, by estimating the minimum cash flow for each year of the project. Then the certainty equivalent cash flows are discounted at the risk-free rate of interest to calculate a Certainty Equivalent NPV. The Certainty Equivalent NPV is then compared with the NPV of the project when its risky cash flows are discounted at the company's required rate of return. If the two NPVs are equivalent, the decision-maker will be indifferent between them and will accept the certain cash flow in place of the risky cash flow. D. The firm's cost of equity capital is not used in calculating certainty equivalent NPV.

Question 26 - CMA 1293 4.11 - Capital Budgeting Methods If an investment project has a profitability index of 1.15, the A. Project's internal rate of return is 15%. B. Project's cost of capital is greater than its internal rate of return. C. Project's internal rate of return exceeds its net present value. D. Net present value of the project is positive. A. The internal rate of return of a project is the discount rate at which a project's net present value is zero. It is not calculated as the profitability index − 1. B. A positive profitability index indicates that the project's profitability is higher than its cost of capital. Therefore, the internal rate of return of the project must be greater than the cost of capital. (c) HOCK international, page 16

Part 2 : Capital Budgeting Methods

C. The internal rate of return is the rate at which a project's net present value is zero. Since the IRR is a rate and the NPV is a monetary amount, the two are not comparable. D. The profitability index for an investment project is its discounted annual net cash inflows divided by its initial cash investment. If a project profitability index is greater than 1.00, we know that its discounted annual net cash inflows are greater than its initial cash investment. Since the net present value is the monetary gain (loss) of the project's cumulative net cash flows, the net present value of a project with a P.I. of greater than 1.00 must be positive.

Question 27 - CMA 1293 4.12 - Capital Budgeting Methods The internal rate of return for a project can be determined A. If the internal rate of return is greater than the firm's cost of capital. B. Only if the project cash flows are constant. C. By finding the discount rate that yields a net present value of zero for the project. D. By subtracting the firm's cost of capital from the project's profitability index. A. The internal rate of return can be determined regardless of whether it is greater, lesser, or the same as the firm's cost of capital. B. While the internal rate of return is easier to calculate when the project's cash flows are constant each year, it is not impossible to calculate it when the project's cash flows vary. When the project's cash flows vary, the internal rate of return can be found by trial and error. C. The internal rate of return is the discount rate which, when used to calculate the net present value of a project, yields a net present value of zero. D. The IRR is not calculated by subtracting the firm's cost of capital from the project's profitability index.

Question 28 - CMA 1293 4.15 - Capital Budgeting Methods A company has unlimited capital funds to invest. The decision rule for the company to follow in order to maximize shareholders' wealth is to invest in all projects having a(n) A. Present value greater than zero. B. Accounting rate of return greater than the hurdle rate used in capital budgeting analyses. C. Net present value greater than zero. D. Internal rate of return greater than zero. A. This is not the description of projects that a company with unlimited capital funds to invest would invest in. B. The accounting rate of return is the ratio of the amount of increased book income to the required investment. Since it uses book income rather than discounted cash flow, it is not comparable to the hurdle rate, which is another term for the firm's required rate of return based on discounted cash flow. C. The net present value of an investment or project is equal to the difference between the present value of all future cash inflows and the present value of the initial and any future cash outflows, using the required rate of return. Thus, in order to maximize shareholders' wealth, a company with unlimited capital funds to invest will invest in all projects having a net present value greater than zero, unless projects are mutually exclusive.

(c) HOCK international, page 17

Part 2 : Capital Budgeting Methods D. A company with unlimited capital to invest would not maximize shareholders' wealth by investing in all projects with an internal rate of return greater than zero. Doing so would fail to consider the company's cost of capital.

Question 29 - CMA 1293 4.16 - Capital Budgeting Methods Sensitivity analysis is used in capital budgeting to A. Simulate probabilistic customer reactions to a new product. B. Estimate a project's internal rate of return. C. Determine the amount that a variable can change without generating unacceptable results. D. Identify the required market share to make a new product viable and produce acceptable results. A. Sensitivity analysis is not used in simulation. B. A project's internal rate of return cannot be estimated by using sensitivity analysis. C. Sensitivity analysis is used to determine how an amount will change if factors that were involved in predicting that amount change. If a small change in the value of one of the inputs causes a large change in the recommended decision, then we say it is sensitive to that input. If we know that a particular input makes a big difference in the analysis, we can take extra care to make sure the value assigned to that input in the analysis is as accurate as possible. Furthermore, the measure of the sensitivity of a project to a change in one of the variables is also an indication of the risk of the project. The more sensitive the project is to a change in one or more variables, the more risky it is. D. Sensitivity analysis cannot be used to identify required market share to make a new product viable.

Question 30 - CMA 1293 4.17 - Capital Budgeting Methods If income tax considerations are ignored, how is depreciation handled by the following capital budgeting techniques? Internal Rate of Return / Accounting Rate of Return / Payback A. Included / Included / Included B. Excluded / Included /Excluded C. Excluded / Excluded / Included D. Included / Excluded / Included A. If income tax considerations are ignored, depreciation would be excluded from the internal rate of return and payback calculations, because the IRR and the payback period are based upon cash flows. B. If income tax considerations are to be ignored, then the depreciation tax shield is ignored. Therefore, the income tax savings from the depreciation are not included in the capital budgeting analyses. If the income tax savings from the depreciation are excluded, then depreciation is ignored in the calculations of internal rate of return and payback. However, depreciation is included in the calculation of the accounting rate of return, because the accounting rate of return is based upon book income, which includes depreciation. C. If income tax considerations are ignored, depreciation would be included in the accounting rate of return calculation, because the accounting rate of return is based upon book income, which includes depreciation. Furthermore, depreciation would be excluded from the payback period calculation, because the payback period calculation is based upon cash flows, not book income. D. If income tax considerations are ignored, depreciation would be excluded from the internal rate of return and payback (c) HOCK international, page 18

Part 2 : Capital Budgeting Methods calculations, because the IRR and the payback period are based upon cash flows. Depreciation would be included in the calculation of the accounting rate of return, because the accounting rate of return is based upon book income, which includes depreciation.

Question 31 - CMA 1293 4.18, adapted - Capital Budgeting Methods The Keego Company is planning a $200,000 equipment investment which has an estimated 5-year life with no estimated salvage value. The company has projected the following annual cash flows for the investment. Year Projected Cash Inflows Present Value of $1 1

$120,000

0.91

2

60,000

0.76

3

40,000

0.63

4

40,000

0.53

5

40,000

0.44

Totals

$300,000

3.27

Assuming that the estimated cash inflows occur evenly during each year, the payback period for the investment is A. 1.50 years. B. 1.67 years. C. 2.50 years. D. 3.94 years. A. 1.5 years results from dividing the total undiscounted cash flows of $300,000 by the initial cash outflow of $200,000. However, this is not the correct way to calculate the payback period. B. A payback period of 1.67 years would result if the second year's cash flow were the same as the first year's cash flow. However, that is not the case. C. The payback period is, first, the number of the project year in the final year when cumulative cash flow (including the initial investment) is negative, plus a fraction consisting of the positive value of the negative cumulative cash inflow amount from the final negative year divided by the cash flow for the following year. In this case, the final year in which the cumulative cash flow is zero is Year 2, because $(200,000) + $120,000 + $60,000 = $(20,000). In the third year, the cash flow is $40,000. So $20,000 ÷ $40,000 = .5, and the payback period is 2 + .5, or 2.5 years. D. This is the discounted payback period, in which all cash flows are discounted and the cumulative discounted cash flow is used to calculate the payback period. Although the discount factors are given in this problem, the problem does not ask for the discounted payback period.

Question 32 - CMA 1293 4.19 - Capital Budgeting Methods The Keego Company is planning a $200,000 equipment investment which has an estimated 5-year life with no estimated salvage value. The company has projected the following annual cash flows for the investment. Year Projected Cash Inflows Present Value of $1 1

$120,000

0.91

(c) HOCK international, page 19

Part 2 : Capital Budgeting Methods 2

60,000

0.76

3

40,000

0.63

4

40,000

0.53

5

40,000

0.44

Totals

$300,000

3.27

The net present value for the investment is A. $100,000 B. $18,800 C. $130,800 D. $218,800 A. $100,000 results from subtracting the initial investment of $200,000 from the total of the undiscounted cash flows, which is $300,000. This is not the correct way to calculate NPV. B. The net present value is the net expected monetary gain or loss from a project when all the expected future cash inflows and outflows are discounted to the point of the investment, using the firm's required rate of return. Discounting the annual cash inflows using the discount factors given results in annual discounted cash inflows of ($120,000 × .91) + ($60,000 × .76) + ($40,000 × .63) + ($40,000 × .53) + ($40,000 × .44) = $218,800. The discounted total annual cash flows minus the initial investment of $200,000 = $18,800, which is the NPV. C. $130,800 results from discounting the initial investment for 5 years and subtracting the discounted value from the present value of the future cash inflows. However, the initial investment which occurs in Year 0 does not need to be discounted in order to calculate net present value, because it is already expressed at its present value in the analysis. D. $218,800 is the total of the present values of the future cash inflows, but this is not net present value.

Question 33 - CMA 1293 4.21 - Capital Budgeting Methods When determining net present value in an inflationary environment, adjustments should be made to A. Decrease the estimated cash inflows and increase the discount rate. B. Increase the estimated cash inflows but not the discount rate. C. Increase the discount rate, only. D. Increase the estimated cash inflows and increase the discount rate. A. The estimated future cash inflows need to be increased, not decreased, to reflect the lower value of the dollar in the future as a result of the inflation. B. The estimated future cash inflows do need to be increased, to reflect the lower value of the dollar in the future as a result of the inflation. However, the discount rate also needs to be increased, because the firm will require a higher rate of return to compensate for the increased inflation. C. The discount rate does need to be increased, because the firm will require a higher rate of return to compensate for the increased inflation. However, this is not the only adjustment that is necessary to determine net present value in an inflationary environment. D. In an environment of inflation, both the discount rate used and the future expected cash flows should be increased. The discount rate is increased because the firm will require a higher rate of return to compensate for the increased inflation. The future expected cash flow amounts need to be increased because inflation will cause the dollar to be worth less in the future, and the amounts of cash (both inflows and outflows) will therefore increase in the future. (c) HOCK international, page 20

Part 2 : Capital Budgeting Methods

Question 34 - CMA 1294 4.20 - Capital Budgeting Methods The length of time required to recover the initial cash outlay of a capital project is determined by using the A. Payback method. B. Weighted net present value method. C. Net present value method. D. Discounted cash flow method. A. The Payback Method is used to determine the number of periods that must pass before the net after-tax cash inflows from the investment will equal (or "pay back") the initial investment cost. If the incoming cash flows are constant over the life of the project, the payback period may be calculated with a simple division as follows: Initial net investment ÷ Periodic constant expected cash flow. If the cash flows are not constant over the life of the project, we must add up the cash inflows and determine on a cumulative basis when the inflows equal the outflows. B. A probability-weighted Net Present Value for a capital project cannot be used to determine the length of time required to recover the initial cash outlay of a capital project. C. The Net Present Value method of capital budgeting analysis cannot be used to determine the length of time required to recover the initial cash outlay of the capital project. D. Methods of capital budgeting analysis that utilize discounted cash flow concepts are Net Present Value (NPV), Internal Rate of Return (IRR), and Profitability Index (PI). NPV can be used to determine the difference between the present value of all future cash inflows and the present value of all (the initial as well as all future) cash outflows, using the required rate of return. A project with a positive NPV is acceptable. The IRR is the discount rate at which the NPV of an investment will be equal to 0, or the discount rate at which the present value of the expected cash inflows from a project equals the present value of the expected cash outflows. If this rate is higher than the required rate of return, the investment is acceptable. The PI calculation is used to determine the ratio of the PV of net future cash flows (both inflows and outflows) to the amount of the initial investment. It is calculated as follows, using the same information from the NPV calculation: PV of future net cash flows ÷ Net Initial Investment. If a project has a positive net present value, the profitability index will be above 1.00 and it will be an acceptable project. However, none of these discounted cash flow methods can be used to determine the length of time required to recover the initial cash outlay of a capital project.

Question 35 - CMA 1294 4.21 - Capital Budgeting Methods In evaluating a capital budget project, the use of the net present value (NPV) model is generally not affected by the A. Initial cost of the project. (c) HOCK international, page 21

Part 2 : Capital Budgeting Methods B. Method of funding the project. C. Type of depreciation used. D. Amount of added working capital needed for operations during the term of the project. A. The initial cost of the project is a component of a net present value capital budgeting analysis. B. The method of funding the project is separate from the net present value method of capital budgeting analysis. C. The type of depreciation used for the project's fixed assets is a component of a net present value capital budgeting analysis. D. Amount of added working capital needed for operations during the term of the project is a component of a net present value capital budgeting analysis.

Question 36 - CMA 1294 4.22 - Capital Budgeting Methods For capital budgeting purposes, management would select a high hurdle rate of return for certain projects because management A. Believes bank loans are riskier than capital investments. B. Wants to factor risk into its consideration of projects. C. Believes too many proposals are being rejected. D. Wants to use equity funding exclusively. A. The method of funding a project is a separate decision from the capital budgeting analysis. B. A company's Weighted Average Cost of Capital (WACC) — which is the rate of return required by investors in the company's securities — is the appropriate discount rate (or hurdle rate) to use in capital budgeting decisions and NPV calculations as long as the riskiness of the project is the same as the riskiness of the firm's existing business. If management wants to factor the risk of a project into its analysis, it will increase the discount rate used in NPV calculations for more risky, or uncertain, investments. A higher discount rate will require higher expected future cash flows in order for the company to make the investment, thus making fewer investments acceptable. C. A high hurdle rate of return would require higher future cash inflows from a project in order for the project to be acceptable. Therefore, a high hurdle rate will result in fewer projects being accepted. D. The method of funding a project is a separate decision from the capital budgeting analysis.

Question 37 - CMA 1294 4.23 - Capital Budgeting Methods The method that recognizes the time value of money by discounting the after-tax cash flows over the life of a project, using the company's minimum desired rate of return is the A. Net present value method. B. Internal rate of return method. C. Accounting rate of return method. D. Payback method. A. The net present value method is used to determine the difference between the present value of all future expected cash inflows and the present value of all (the initial as well as all future) expected cash outflows, (c) HOCK international, page 22

Part 2 : Capital Budgeting Methods using the required rate of return. A project with a positive NPV is acceptable. B. The internal rate of return is the discount rate at which the NPV of an investment will be equal to 0, or the discount rate at which the present value of the expected cash inflows from a project equals the present value of the expected cash outflows. The company's minimum desired rate of return is used only in analyzing the results of the IRR analysis, not in doing the calculation. If the IRR is higher than the company's minimum desired rate of return, the project is acceptable. C. The accounting rate of return is a ratio of the amount of increased book income to the required investment. It does not recognize the time value of money and it does not include discounting after-tax cash flows over the life of the project. D. The payback method does not recognize the time value of money and does not include discounting after-tax cash flows over the life of the project.

Question 38 - CMA 1294 4.24 - Capital Budgeting Methods The method that divides a project's annual after-tax net income by the average investment cost to measure the estimated performance of a capital investment is the A. Payback method. B. Accounting rate of return method. C. Internal rate of return method. D. Net present value (NPV) method. A. The Payback Method is used to determine the number of periods that must pass before the net after-tax cash inflows from the investment will equal (or "pay back") the initial investment cost. If the incoming cash flows are constant over the life of the project, the payback period may be calculated with a simple division as follows: Initial net investment Periodic constant expected cash flow If the cash flows are not constant over the life of the project, the Payback Period is calculated by adding up the cash inflows and determining on a cumulative basis when the inflows equal the outflows. The Payback Method does not utilize a division of the project's annual after-tax net income by the average investment cost to measure the estimated performance of a capital investment. B. The accounting rate of return is a ratio of the amount of increased book income to the required investment. It is calculated as follows: Increase in Expected Annual Average After Tax Accounting Net Income ÷ Net Initial Investment. Sometimes the average investment figure is used rather than the total investment. Since this method uses accrual accounting income, it includes depreciation. However, it does not take into account the time value of money. C. The internal rate of return is the discount rate at which the NPV of an investment will be equal to 0, or the discount rate at which the present value of the expected cash inflows from a project equals the present value of the expected cash outflows. If the IRR is higher than the company's minimum desired rate of return, the project is acceptable. The IRR does not utilize a division of the project's annual after-tax net income by the average investment cost to measure the estimated performance of a capital investment. D. The net present value method is used to determine the difference between the present value of all future cash inflows and the present value of all (the initial as well as all future) cash outflows, using the required rate of return. A project with a positive NPV is acceptable. It does not utilize a division of the project's annual after-tax net income by the average investment cost to measure the estimated performance of a capital investment.

Question 39 - CMA 1294 4.25 - Capital Budgeting Methods (c) HOCK international, page 23

Part 2 : Capital Budgeting Methods

The capital budgeting model that is generally considered the best model for long-range decision making is the A. Unadjusted rate of return model. B. Accounting rate of return model. C. Discounted cash flow model. D. Payback model. A. The unadjusted rate of return model, or accounting rate of return model, is not generally considered the best model for long-range decision making, because it does not incorporate time value of money concepts. B. The accounting rate of return model, or unadjusted rate of return model, is not generally considered the best model for long-range decision making, because it does not incorporate time value of money concepts. C. Discounted cash flow methods of capital budgeting, including net present value, internal rate of return, and profitability index, are generally considered the best model for long-range capital budgeting decision making. D. The payback method is not generally considered the best model for long-range decision making, because it does not incorporate time value of money concepts.

Question 40 - CMA 1294 4.26 - Capital Budgeting Methods The technique used to evaluate all possible capital projects of different dollar amounts and then rank them according to their desirability is the A. Discounted cash flow method. B. Payback method. C. Profitability index method. D. Net present value method. A. There are three discounted cash flow methods used for capital budgeting. Only one of them can be used to evaluate all possible capital projects of different dollar amounts and rank them according to their desirability. B. The Payback Method is used to determine the number of periods that must pass before the net after-tax cash inflows from the investment will equal (or "pay back") the initial investment cost. It is not used to evaluate all possible capital projects of different dollar amounts and then rank them according to their desirability. C. The PI calculation is used to determine the ratio of the PV of net future cash flows (both inflows and outflows) to the amount of the initial investment. It is calculated as follows, using the same information from the NPV calculation: PV of future net cash flows ÷ Net Initial Investment. If a project has a positive net present value, the profitability index will be above 1.00 and it will be an acceptable project. The profitability index is used to evaluate all possible capital projects of different dollar amounts and then rank them according to their desirability. D. The net present value method is not used to evaluate all possible capital projects of different dollar amounts and then rank them according to their desirability.

Question 41 - CMA 1294 4.27 - Capital Budgeting Methods A widely used approach that is used to recognize uncertainty about individual economic variables while obtaining an immediate financial estimate of the consequences of possible prediction errors is

(c) HOCK international, page 24

Part 2 : Capital Budgeting Methods A. Expected value analysis. B. Sensitivity analysis. C. Learning curve analysis. D. Regression analysis. A. Expected value is the weighted average of all the possible values of a random variable, with the probabilities of each of the values used as the weights. The expected value is the mean value, also known as the average value. Expected value analysis does not yield an immediate financial estimate of the consequences of possible prediction errors. B. Sensitivity analysis can be used to determine how cash flows can be expected to vary with changes in the underlying assumptions. Using expected cash flows, the NPV, IRR, and PI of the project are determined. Then, the key assumptions that were used in making the original expected cash flow projections are identified. One assumption at a time is then changed, leaving the other assumptions unchanged, and the NPV, IRR and PI are recalculated to determine what effect changing one assumption would have on those measures. C. Learning curves describe the fact that the more experience people have with something, the more efficient they become in doing that task. Higher costs per unit early in production are part of start-up costs. It is commonly accepted that new products and production processes experience a period of low productivity followed by increased productivity. However, learning curve analysis does not provide an immediate financial estimate of the consequences of possible prediction errors. D. Regression analysis is a method of forecasting, using trend analysis. However, it does not yield an immediate financial estimate of the consequences of possible prediction errors.

Question 42 - CMA 1295 4.1 - Capital Budgeting Methods Which one of the following statements about the payback method of investment analysis is correct? The payback method A. Considers cash flows after the payback has been reached. B. Does not consider the time value of money. C. Uses discounted cash flow techniques. D. Generally leads to the same decision as other methods for long-term projects. A. The payback method does not take into account any cash flows expected to be received after the payback point has been reached. B. The payback method uses undiscounted cash flows and thus does not incorporate the time value of money in the analysis. That is one of its weaknesses. Another weakness is that it does not take into account any cash flows that are received after the payback point has been reached. C. The payback method does not use discounted cash flow techniques, and that is one of its weaknesses. D. The payback method does not necessarily lead to the same decision as other methods of analyzing long-term projects do.

Question 43 - CMA 1295 4.12 (adapted) - Capital Budgeting Methods Willis Inc. has a cost of capital of 15% and is considering the acquisition of a new machine which costs $400,000 and has a useful life of 5 years. Willis projects that earnings and cash flow will increase as follows:

(c) HOCK international, page 25

Part 2 : Capital Budgeting Methods

Net Year 1 2 3 4 5

Period 1 2 3 4 5

After-Tax Earnings $100,000 100,000 100,000 100,000 200,000

Cash Flow $160,000 140,000 100,000 100,000 100,000

15% Interest Rate factors Present Value of an Present Value of $1 Annuity of $1 0.87 0.87 0.76 1.63 0.66 2.29 0.57 2.86 0.50 3.36

The net present value of this investment is A. Negative, $14,000. B. Positive, $200,000. C. Negative, $64,000. D. Positive, $18,600. A. An answer of a negative $14,000 results from using the net earnings amounts instead of the after-tax cash flow amounts in the net present value analysis. B. An answer of $200,000 results from subtracting the initial investment from the total of the undiscounted cash inflows. C. An answer of a negative $64,000 NPV results from using annual cash flows of $100,000, and forgetting to discount the additional cash flows of $60,000 in Year 1 and $40,000 in Year 2. D. The net present value is the present value of all cash flows after Year 0 (positive and negative) less the initial investment. To calculate the present value of the cash flows after year 0, you could discount each individual cash flow amount by the appropriate present value of $1 factor. However, that is not the most time-effective way to do it. If you recognize that all the annual cash flow amounts contain an amount such as $100,000 and $100,000 is the exact amount of the cash flow for at least two of the years, you can save time by calculating first the present value of an annuity for the $100,000; then calculating the present value of $1 individually for any amounts over the $100,000 amount. In this case, we have 5 years of $100,000 cash flows, and the discount factor given for the PV of an annuity for 5 years is 3.36. We also have $60,000 to be discounted for one year and $40,000 to be discounted for two years using the appropriate present value of $1 factors. Thus, the present value of the cash inflows is ($100,000 × 3.36) + ($60,000 × .87) + ($40,000 × .76) = $418,600. The net present value is $418,600 less the initial investment of $400,000, or $18,600.

Question 44 - CMA 1295 4.13 (adapted) - Capital Budgeting Methods Willis Inc. has a cost of capital of 15% and is considering the acquisition of a new machine which costs $400,000 and has a useful life of 5 years. Willis projects that earnings and cash flow will increase as follows:

(c) HOCK international, page 26

Part 2 : Capital Budgeting Methods After-Tax Earnings $100,000 100,000 100,000 100,000 200,000

Net Year 1 2 3 4 5

Period 1 2 3 4 5

Cash Flow $160,000 140,000 100,000 100,000 100,000

15% Interest Rate factors Present Value of an Present Value of $1 Annuity of $1 0.87 0.87 0.76 1.63 0.66 2.29 0.57 2.86 0.50 3.36

What is the payback period of this investment? A. 4.00 years. B. 3.00 years. C. 1.50 years. D. 4.63 years. A. An answer of 4.00 years results from using net earnings instead of after-tax cash flows to calculate the payback period. B. The payback period is the number of periods that must pass before the net after-tax cash inflows from the investment will equal (or "pay back") the initial investment. The initial investment of $400,000 is returned in exactly 3 years, because the cash inflows of Years 1 through 3 -- $160,000, $140,000, and $100,000 -- total $400,000. C. The payback method is used to determine the number of periods that must pass before the net after-tax cash inflows from the investment will equal (or "pay back") the initial investment cost. After 1.5 years has passed, only $230,000 of the initial $400,000 investment will have been paid back. D. 4.63 years is the Discounted Payback Period (also called breakeven time) which results from calculating the payback period using discounted cash flows. However, the Payback Method does not use discounted cash flows.

Question 45 - CMA 1295 4.14 - Capital Budgeting Methods The net present value of a proposed investment is negative; therefore, the discount rate used must be A. Less than the risk-free rate. B. Greater than the project's internal rate of return. C. Less than the project's internal rate of return. D. Greater than the firm's cost of equity. A. The risk-free rate is not relevant to a capital budgeting analysis using net present value. B. The internal rate of return is the discount rate at which a project's net present value is zero. As the discount (c) HOCK international, page 27

Part 2 : Capital Budgeting Methods rate used increases, the net present value of a project decreases. Therefore, if the discount rate used is higher than the project's internal rate of return, the net present value of the project will be less than zero and thus will be a negative amount. C. The internal rate of return is the discount rate at which a project's net present value is zero. As the discount rate used decreases, the net present value of a project increases. Therefore, if the discount rate used is lower than the project's internal rate of return, the net present value of the project will be greater than zero. D. Use of a discount rate that is greater than the firm's cost of equity may result in a negative net present value for the investment, or it may result in a positive net present value for the investment. Which it will be depends upon the cash flows.

Question 46 - CMA 1295 4.16 - Capital Budgeting Methods A disadvantage of the net present value method of capital expenditure evaluation is that it A. Computes the true interest rate. B. Is calculated using sensitivity analysis. C. Is difficult to apply because it uses a trial-and-error approach. D. Does not provide the true rate of return on investment. A. NPV analysis does not compute a true interest rate on an investment. B. Sensitivity analysis, which is a "what if" technique, can be used with NPV analysis to determine how cash flows and thus NPV can be expected to vary with changes in the underlying assumptions. C. NPV analysis does not use a trial-and-error approach. D. NPV analysis provides a dollar amount by which the present value of the return on a project is greater than the investment. It does not provide an actual rate of return for the investment.

Question 47 - CMA 1295 4.2 - Capital Budgeting Methods Barker Inc. has no capital rationing constraint and is analyzing many independent investment alternatives. Barker should accept all investment proposals A. That have positive cash flows. B. If debt financing is available for them. C. That provide returns greater than the before-tax cost of debt. D. That have a positive net present value. A. A project might have positive cash flows but have a negative net present value, in which case it would not be a good investment. B. The availability of financing is a completely separate issue from the capital budgeting process. C. Since a project may be financed with any combination of debt and equity, it is not appropriate to compare the expected return from the project to the before-tax cost of debt. D. If a company has no restrictions on its available capital for investment, and if all the projects under consideration are independent (i.e., none of the projects are mutually exclusive, meaning if the company invests in one it cannot invest in another for reasons other than capital availability), then the company should accept all (c) HOCK international, page 28

Part 2 : Capital Budgeting Methods projects with a positive net present value.

Question 48 - CMA 1295 4.6 - Capital Budgeting Methods When the risks of the individual components of a project's cash flows are different, an acceptable procedure to evaluate these cash flows is to A. Divide each cash flow by the payback period. B. Discount each cash flow using a discount rate that reflects the degree of risk. C. Compute the net present value of each cash flow using the firm's cost of capital. D. Compare the internal rate of return from each cash flow to its risk. A. The payback period is not relevant to evaluating risk in cash flows. B. A company's Weighted Average Cost of Capital (WACC) — which is the rate of return required by investors in the company's securities — is the appropriate discount rate to use in capital budgeting decisions and NPV calculations as long as the riskiness of the project is the same as the riskiness of the firm's existing business. When the riskiness of the project is different from that of the company's existing business, the discount rate used to calculate NPV needs to be adjusted to reflect the changed risk profile of the firm as a result of the project under consideration. A higher discount rate is used to reflect higher risk; a lower discount rate reflects lower risk. And when the risks of the individual components of a project's cash flows are different, it is appropriate to use a risk-adjusted discount rate for each component that is specific for the degree of risk inherent in that component of the cash flow. C. The firm's cost of capital is the appropriate discount rate to use in capital budgeting decisions and NPV calculations only if the riskiness of the project is the same as the riskiness of the firm's existing business. When the riskiness of the project or of any individual component of the project's cash flow is different from that of the company's existing business, the discount rate used to calculate NPV needs to be adjusted to reflect the changed risk profile of the firm as a result of the project under consideration. D. The internal rate of return is the discount rate at which the NPV is zero. It is not used in evaluating risk.

Question 49 - CMA 1295 4.7 - Capital Budgeting Methods The NPV of a project has been calculated to be $215,000. Which one of the following changes in assumptions would decrease the NPV? A. Decrease the initial investment amount. B. Increase the discount rate. C. Extend the project life and associated cash inflows. D. Decrease the estimated effective income tax rate. A. Decreasing the initial investment amount will increase the NPV. B. Increasing the discount rate will decrease the present value of the cash inflows, which will decrease the NPV. C. Extending the life of the project will increase the cash flows and thus increase the NPV. D. Decreasing the estimated effective income tax rate will increase the net of tax cash inflows, which will increase the NPV.

(c) HOCK international, page 29

Part 2 : Capital Budgeting Methods

Question 50 - CMA 1295 4.9 - Capital Budgeting Methods The net present value method of capital budgeting assumes that cash flows are reinvested at A. The rate of return of the project. B. The discount rate used in the analysis. C. The risk-free rate. D. The cost of debt. A. The internal rate of return method of analyzing a capital investment project assumes that the resulting cash flows will be able to be reinvested at the rate of return of the project, but the net present value method does not. B. The net present value method of capital budgeting involves the assumption that the resulting cash flows will be able to be invested at the rate of return that is used as a discount rate in the analysis. C. The risk-free rate is not a part of net present value analysis of a capital budgeting project. D. The cost of debt for a capital investment is not the rate that the resulting cash flows are assumed to be reinvested at in net present value analysis.

Question 51 - CMA 691 4.17 - Capital Budgeting Methods Capital budgeting methods are often divided into two classifications: project screening and project ranking. Which one of the following is considered a ranking method rather than a screening method? A. Time-adjusted rate of return. B. Accounting rate of return. C. Profitability index. D. Net present value. A. "Time-adjusted rate of return" is another term that is used to refer to the internal rate of return. The internal rate of return is a project screening method, not a ranking method. If the internal rate of return of a project is equal to or higher than the required rate of return, then the project is acceptable. However, there are problems associated with trying to use the internal rate of return as a ranking method, so it is not used in that way. B. The accounting rate of return is not a ranking method of capital budgeting analysis. C. The profitability index is a benefit/cost ratio. It represents the ratio of the benefits (net cash inflows) to the costs (net initial investment). The profitability index enables us to compare, or rank, the benefit/cost ratios of different sized investments, since the profitability index expresses profitability on a percentage basis rather than a total dollar amount basis. It is very useful when we must compare multiple investments that are of different investment amounts, the projects are not mutually exclusive, and we need to rank them. When there are multiple investment opportunities, we will take the project that has the highest profitability index. When the projects are independent alternatives of the same length, this method will give us the same accept/reject decisions that NPV provides. However, when the projects are of different time periods or amounts of initial investments, the profitability index may give different rankings than NPV. D. Net present value is primarily a screening method (although it may be used as a ranking method in certain circumstances). The general rule is that any project that has a positive NPV should be accepted, while a project with a negative NPV should be rejected. However, for various reasons such as limited funds to invest or nonfinancial factors, not all projects with positive NPVs will be acceptable. It is probably better to state that any project that has a positive NPV is a candidate for further consideration - it has passed the first screen. A project with a negative NPV should not be considered further. This will maximize shareholder wealth. When there are limited funds to invest, NPV is useful (c) HOCK international, page 30

Part 2 : Capital Budgeting Methods because it enables ranking of the various projects according to the amount that they are expected to return. However, it is not primarily a ranking method rather than a screening method.

Question 52 - CMA 691 4.18 - Capital Budgeting Methods The accounting rate of return A. Is synonymous with the internal rate of return. B. Is inconsistent with the divisional performance measure known as return on investment. C. Recognizes the time value of money. D. Focuses on income as opposed to cash flows. A. The accounting rate of return is very different from the internal rate of return. The internal rate of return is the discount rate at which the net present value of an investment is zero. The accounting rate of return does not incorporate present value concepts at all. B. The accounting rate of return is not inconsistent with the divisional performance measure known as Return On Investment (ROI). In fact, the accounting rate of return is quite consistent with Return On Investment. C. The accounting rate of return does not take into account the time value of money. D. The accounting rate of return uses accrual accounting income, including depreciation, rather than cash flows. It does not take into account the time value of money, and for that reason it is also called the unadjusted rate of return model.

Question 53 - CMA 691 4.19 - Capital Budgeting Methods The internal rate of return on an investment A. Would tend to be reduced if a company used an accelerated method of depreciation for tax purposes rather than the straight-line method. B. May produce different rankings from the net present value method on mutually exclusive projects. C. Disregards discounted cash flows. D. Usually coincides with the company's hurdle rate. A. The internal rate of return on a project would tend to be increased if a company used an accelerated method of depreciation for tax purposes rather than the straight-line method, because an accelerated method of depreciation would result in a higher depreciation tax shield in the early years of the investment. This in turn would lead to a higher present value for the future expected cash flows, which would result in an increased internal rate of return. B. If two investments are mutually exclusive, then accepting one means we cannot accept the other. This can occur if, for example, the cash flows of one will be adversely impacted by the acceptance of the other; or if the use of land for one project precludes its use for another project. When mutually exclusive investments are being considered, there may be a conflict between the NPV results and the IRR results in determining which of the two mutually exclusive projects should be accepted. One may have a higher IRR, while the other has a higher NPV. C. The internal rate of return is the discount rate at which the NPV of a project is equal to zero. Thus, it does not disregard discounted cash flows. D. The internal rate of return is compared with the company's hurdle rate (i.e., its required rate of return) in determining whether a project is worthwhile. However, it would be strictly coincidental if the internal rate of return on a given project (c) HOCK international, page 31

Part 2 : Capital Budgeting Methods were to coincide exactly with the company's hurdle rate.

Question 54 - CMA 691 4.21 - Capital Budgeting Methods On January 1, Crane Company will acquire a new asset that costs $400,000 and is anticipated to have a salvage value of $30,000 at the end of 4 years. The new asset Qualifies as 3-year property under the Modified Accelerated Cost Recovery System (MACRS). Will replace an old asset that currently has a tax basis of $80,000 and can be sold now for $60,000. Will continue to generate the same operating revenues as the old asset ($200,000 per year). However, savings in operating costs will be experienced as follows: a total of $120,000 in each of the first 3 years and $90,000 in the fourth year. Crane is subject to a 40% tax rate and rounds all computations to the nearest dollar. Assume that any gain or loss affects the taxes paid at the end of the year in which it occurred. The company uses the net present value method to analyze projects using the following factors and rates: Present Value Present Value of $1 at 14% Period of $1 at 14% annuity MACRS 1

.88

.88

33%

2

.77

1.65

45

3

.68

2.33

15

4

.59

2.92

7

The present value of the depreciation tax shield for the fourth year MACRS depreciation of Crane Company's new asset is A. $16,520. B. $6,112. C. $6,608. D. $0. A. An answer of $16,520 results from discounting the fourth year's depreciation amount to year 0. The question asks for the present value of the fourth year's depreciation tax shield, not the present value of the fourth year's depreciation. B. An answer of $6,112 results from using the gross asset cost less the salvage value to calculate the depreciation. However, under MACRS, the depreciation rate is applied to the gross asset cost, not the depreciable base. C. Although this question involves replacing one asset with another asset, we are not given information that would enable us to do an incremental analysis of the difference (if any) between the depreciation on the new asset versus the depreciation on the old asset. Thus, we will analyze this as if it were a new asset purchase, not a replacement of an existing asset. Since depreciation under MACRS is applied to the gross purchase price of the asset ($400,000) and the fourth year's depreciation will be 7% of $400,000, the fourth year's depreciation will be $28,000. Crane's tax rate is 40%, so the depreciation tax shield is $28,000 × .40, or $11,200. Since the question asks for the present value of the depreciation tax shield for the fourth year, we will discount it using .59 from the table (the PV of $1 at 14% for 4 years). $11,200 × .59 = $6,608, which is the present value of the depreciation tax shield for the fourth year of MACRS depreciation. D. An answer of $0 results from failing to recognize that under MACRS, the "half-year convention" is normally used. Under the half-year convention, one-half of one year's depreciation is taken in the first year the asset is placed in service. For an asset with a three-year life, one year's depreciation would then be taken in each of years 2 and 3; and one-half of one year's depreciation would be taken in year 4. Thus, the present value of the depreciation tax shield for the fourth (c) HOCK international, page 32

Part 2 : Capital Budgeting Methods year could not be zero because the depreciation in year 4 would not be zero.

Question 55 - CMA 691 4.22 - Capital Budgeting Methods On January 1, Crane Company will acquire a new asset that costs $400,000 and is anticipated to have a salvage value of $30,000 at the end of 4 years. The new asset Qualifies as 3-year property under the Modified Accelerated Cost Recovery System (MACRS). Will replace an old asset that currently has a tax basis of $80,000 and can be sold now for $60,000. Will continue to generate the same operating revenues as the old asset ($200,000 per year). However, savings in operating costs will be experienced as follows: a total of $120,000 in each of the first 3 years and $90,000 in the fourth year. Crane is subject to a 40% tax rate and rounds all computations to the nearest dollar. Assume that any gain or loss affects the taxes paid at the end of the year in which it occurred. The company uses the net present value method to analyze projects using the following factors and rates: Present Value Present Value of $1 at 14% Period of $1 at 14% annuity MACRS 1

.88

.88

33%

2

.77

1.65

45

3

.68

2.33

15

4

.59

2.92

7

The discounted net-of-tax amount that should be factored into Crane Company's analysis for the disposal transaction is A. $60,000. B. $67,040. C. $68,000. D. $45,760. A. $60,000 is the cash flow from the disposition of the old asset, but that is not the only component of the discounted net-of-tax amount that should be factored into Crane Company's analysis for the disposal transaction. The tax savings from the loss on the disposition must also be considered. B. We are told in the question, "Assume that any gain or loss affects the taxes paid at the end of the year in which it occurred." Therefore, the tax savings that results from the $20,000 loss on the sale of the old equipment that occurs in year 0 must be discounted for one year. The tax savings is $8,000 ($20,000 × .40). Discounted for one year at 14%, it is $8,000 × .88, or $7,040. Cash flow from the disposal in year 0 is $60,000, and no discounting is necessary. Thus, the discounted net of tax amount to factor into Crane Company's analysis for the disposal transaction is $60,000 + $7,040, or $67,040. C. An answer of $68,000 results from using the tax savings from the loss on the disposition at its gross, undiscounted value. We are told in the question, "Assume that any gain or loss affects the taxes paid at the end of the year in which it occurred." Therefore, the tax savings from the loss on the disposition should be discounted for one year. D. An answer of $45,760 results from two errors: One, discounting the $60,000 cash that is received in year 0 for one year, which is incorrect because the cash received in year 0 needs no discounting. And two, the answer results from subtracting the present value of the tax savings from the present value of the cash received in the disposition, which is incorrect. The tax savings is an increase to cash, not a decrease.

(c) HOCK international, page 33

Part 2 : Capital Budgeting Methods

Question 56 - CMA 692 4.15 - Capital Budgeting Methods The recommended technique for evaluating projects when capital is rationed and there are no mutually exclusive projects from which to choose is to rank the projects by A. Internal rate of return. B. Profitability index. C. Payback. D. Accounting rate of return. A. The IRR is the discount rate at which the NPV of an investment will be equal to 0, i.e., the discount rate at which the present value of the expected cash inflows from a project equals the present value of the expected cash outflows. If this rate is higher than the required rate of return, the investment is acceptable. If this rate is lower than the required rate of return, the investment should not be made. IRR is a good screening method, meaning it is useful for evaluating whether or not an investment should be made. However, it is not useful for ranking projects when capital is rationed, because it cannot be used to determine which project(s) among all of the acceptable projects will be most profitable. A project with a very small initial investment may have a high IRR, but its NPV may be lower than the NPV of a project requiring a larger initial investment. B. When capital is limited and the projects from which to choose are not mutually exclusive, the decision as to which project should receive money first is made using the Profitability Indexes of the different proposed projects. The Profitability Index is a variation of the benefit/cost ratio. It represents the ratio of the benefits (present value of net cash inflows) to the costs (net initial investment). The Profitability Index enables us to compare (or rank) the benefit/cost ratios of different sized investments, since the Profitability Index expresses profitability on a percentage basis rather than a total dollar amount basis. It is very useful when we must compare multiple investments that are of different investment amounts, the projects are not mutually exclusive, and we need to prioritize them. C. The Payback Method is used to determine the number of periods that must pass before the net after-tax cash inflows from the investment will equal (or "pay back") the initial investment cost. The Payback Method is a screening method of capital budgeting analysis, meaning that it can be used to determine whether an investment is worthwhile or not. However, it is not a means to rank acceptable projects. Furthermore, the Payback Period does not use discounted cash flow techniques, and it ignores any profits from the project that occur after the initial investment has been recouped. D. The Accounting Rate of Return is a ratio of the amount of increased book income to the required investment. Since this method uses accrual accounting income, it includes depreciation. It does not take into account the time value of money, so it is less useful than the discounted cash flow methods of capital budgeting.

Question 57 - CMA 692 4.16 - Capital Budgeting Methods The net present value (NPV) method of investment project analysis assumes that the project's cash flows are reinvested at the A. Risk-free interest rate. B. Firm's accounting rate of return. C. Computed internal rate of return. D. Discount rate used in the NPV calculation. A. NPV calculation does not assume that cash inflows from the project will be reinvested at the risk-free interest rate. B. The Accounting Rate of Return is a ratio of the amount of increased book income to the required investment. NPV calculation does not assume that cash inflows from the project will be reinvested at the Accounting Rate of Return.

(c) HOCK international, page 34

Part 2 : Capital Budgeting Methods C. The Internal Rate of Return is the discount rate at which NPV is zero. NPV calculation does not assume that cash inflows from the project will be reinvested at the Internal Rate of Return. D. The Net Present Value (NPV) method calculates the expected monetary gain or loss from a project by discounting all expected future cash inflows and outflows to the present point in time, using the required rate of return. This use of a required rate of return in discounting assumes that all the future cash inflows from the project will be able to be reinvested at the same required rate of return, which may not be the situation.

Question 58 - CMA 692 4.21 - Capital Budgeting Methods The technique that measures the estimated performance of a capital investment by dividing the project's annual after-tax net income by the average investment cost is called the A. Accounting rate of return method. B. Internal rate of return method. C. Bail-out payback method. D. Profitability index method. A. The accounting rate of return method is a ratio of the amount of increased book income as a result of the project under consideration to the required investment. It is calculated as: Increase in Expected Annual Average After Tax Accounting Net Income ÷ Net Initial Investment OR Average Investment. B. The Internal Rate of Return is the discount rate at which a project's NPV is zero. C. The bailout payback method is calculated in the same manner as the payback method, but recognition of the possibility that the project may be ended prematurely and the equipment sold is added to the calculations. Any salvage value (after-tax) of the equipment at various dates is added to the cash inflows of the project through the same dates. The cash flows that would result upon termination of the project at various dates are included in the payback period calculation. Use of bailout payback offers the best protection if things go wrong, because it shows what the effect would be of "bailing out" of the project. D. The Profitability Index is a benefit/cost ratio. It represents the ratio of the benefits (present value of the net cash inflows) to the costs (net initial investment). The Profitability Index enables us to compare (or rank) the benefit/cost ratios of different sized investments, since the Profitability Index expresses profitability on a percentage basis rather than a total dollar amount basis.

Question 59 - CMA 693 4.20 - Capital Budgeting Methods Amster Corporation has not yet decided on its hurdle rate for use in the evaluation of capital budgeting projects. This lack of information will prohibit Amster from calculating a project's Accounting Net Internal Rate of Return Present Value Rate of Return

A. No Yes No B. Yes Yes Yes C. No Yes Yes D. No No No

(c) HOCK international, page 35

Part 2 : Capital Budgeting Methods A. Not knowing the hurdle rate will not prevent the calculation of the Accounting Rate of Return or the Internal Rate of Return. However, it will prohibit the company from calculating NPV, because in order to calculate NPV, the future cash flows need to be discounted using the hurdle rate. B. The Accounting Rate of Return and the Internal Rate of Return can be calculated without knowing the hurdle rate. C. Internal Rate of Return can be calculated without knowing the hurdle rate, because the IRR is the discount rate that causes NPV to be zero. D. NPV cannot be calculated unless the hurdle rate is first determined, because in order to calculate NPV, the future cash flows need to be discounted using the hurdle rate.

Question 60 - CMA 693 4.22 - Capital Budgeting Methods The following selected data pertain to a 4-year project being considered by Metro Industries: • A depreciable asset that costs $1,200,000 will be acquired on January 1. The asset, which is expected to have a $200,000 salvage value at the end of 4 years, qualifies as 3-year property under the Modified Accelerated Cost Recovery System (MACRS). • The new asset will replace an existing asset that has a tax basis of $150,000 and can be sold on the same January 1 for $180,000. • The project is expected to provide added annual sales of 30,000 units at $20. Additional cash operating costs are: variable, $12 per unit; fixed, $90,000 per year. • A $50,000 working capital investment that is fully recoverable at the end of the fourth year is required. Metro is subject to a 40% income tax rate and rounds all computations to the nearest dollar. Assume that any gain or loss affects the taxes paid at the end of the year in which it occurred. The company uses the net present value method to analyze investments and will employ the following factors and rates.

Period

Present Value of Present Value of $1 at 12% $1 Annuity at 12% MACRS

1

0.89

0.89

33%

2

0.80

1.69

45

3

0.71

2.40

15

4

0.64

3.04

7

The discounted cash flow for the fourth year MACRS depreciation on the new asset is A. $17,920. B. $0. C. $26,880. D. $21,504. A. Under MACRS depreciation using the half-year convention, 100% of an asset's cost is depreciated. A fourth year cash flow of $17,920 results from subtracting the salvage value from the cost to determine the depreciable base. However, under MACRS, salvage value is not used because 100% of the asset's cost is depreciated. B. Under MACRS depreciation using the half-year convention, 100% of an asset's cost is depreciated. One-half of one year's depreciation is taken in the first year the asset is owned, one-half of one year's depreciation is taken in the last year it is depreciated, and one year's depreciation is taken in each of the other years of the asset's life. According to the (c) HOCK international, page 36

Part 2 : Capital Budgeting Methods table given, 7% of the cost will be taken as depreciation in the fourth year. Therefore, there will be a discounted cash flow in the fourth year as a result of the depreciation tax shield. C. Under MACRS depreciation using the half-year convention, 100% of an asset's cost is depreciated. A fourth year cash flow of $26,880 results from subtracting the salvage value from the cost to determine the depreciable base and also from using a 60% tax rate before discounting the cash flow for four years at 12%. The salvage value should not be subtracted, and the tax rate used should be 40%. D. Under MACRS depreciation using the half-year convention, 100% of an asset's cost is depreciated. One-half of one year's depreciation is taken in the first year the asset is owned, one-half of one year's depreciation is taken in the last year it is depreciated, and one year's depreciation is taken in each of the other years of the asset's life. According to the table given, 7% of the cost will be taken as depreciation in the fourth year. Since the asset cost $1,200,000, 7% of that is equal to $84,000. With a 40% tax rate, $84,000 × .40, or $33,600 of other income will be shielded from taxation by the depreciation in the fourth year. The present value of $33,600 in four years, discounted at 12%, is $33,600 × .64, or $21,504.

Question 61 - CMA 693 4.23 - Capital Budgeting Methods The following selected data pertain to a 4-year project being considered by Metro Industries: • A depreciable asset that costs $1,200,000 will be acquired on January 1. The asset, which is expected to have a $200,000 salvage value at the end of 4 years, qualifies as 3-year property under the Modified Accelerated Cost Recovery System (MACRS). • The new asset will replace an existing asset that has a tax basis of $150,000 and can be sold on the same January 1 for $180,000. • The project is expected to provide added annual sales of 30,000 units at $20. Additional cash operating costs are: variable, $12 per unit; fixed, $90,000 per year. • A $50,000 working capital investment that is fully recoverable at the end of the fourth year is required. Metro is subject to a 40% income tax rate and rounds all computations to the nearest dollar. Assume that any gain or loss affects the taxes paid at the end of the year in which it occurred. The company uses the net present value method to analyze investments and will employ the following factors and rates.

Period

Present Value of Present Value of $1 at 12% $1 Annuity at 12% MACRS

1

0.89

0.89

33%

2

0.80

1.69

45

3

0.71

2.40

15

4

0.64

3.04

7

The discounted, net-of-tax amount that relates to disposal of the existing asset is A. $180,000. B. $168,000. C. $169,320. D. $190,680. A. $180,000 is the amount of cash to be received for the existing asset, but it does not include the effect of taxes due on the gain. It also is not discounted.

(c) HOCK international, page 37

Part 2 : Capital Budgeting Methods B. $168,000 is the amount of cash to be received for the existing asset less the amount of tax liability for the gain. However, the question asks for the discounted, net-of-tax amount that relates to the disposal of the existing asset. In order to find the discounted amount, it is necessary to discount the income tax liability since the tax is not due until year end. C. Cash received from the sale of the existing asset on January 1 will be $180,000. The tax basis of the asset on that date will be $150,000. Therefore, income tax will be due on the gain of $30,000 at the rate of 40%, so income tax will be $12,000. However, the problem states that any gain or loss affects the taxes paid at the end of the year in which it occurred. Therefore, tax on the January 1 gain will not be due until year end. Thus, we discount the $12,000 income tax liability for one year at the 12% rate given in the Present Value of $1 factor table. $12,000 × .89 = $10,680, so the present value of the income tax liability is $10,680. The discounted, net-of-tax amount that relates to disposal of the existing asset is $180,000 − $10,680, which equals $169,320. D. $190,680 results from adding the discounted tax liability to the cash inflow from the sale of the existing asset. The discounted tax liability should be a deduction to the cash inflow from the sale of the existing asset, not an addition to it.

Question 62 - CMA 693 4.24 - Capital Budgeting Methods The following selected data pertain to a 4-year project being considered by Metro Industries: • A depreciable asset that costs $1,200,000 will be acquired on January 1. The asset, which is expected to have a $200,000 salvage value at the end of 4 years, qualifies as 3-year property under the Modified Accelerated Cost Recovery System (MACRS). • The new asset will replace an existing asset that has a tax basis of $150,000 and can be sold on the same January 1 for $180,000. • The project is expected to provide added annual sales of 30,000 units at $20. Additional cash operating costs are: variable, $12 per unit; fixed, $90,000 per year. • A $50,000 working capital investment that is fully recoverable at the end of the fourth year is required. Metro is subject to a 40% income tax rate and rounds all computations to the nearest dollar. Assume that any gain or loss affects the taxes paid at the end of the year in which it occurred. The company uses the net present value method to analyze investments and will employ the following factors and rates.

Period

Present Value of Present Value of $1 at 12% $1 Annuity at 12% MACRS

1

0.89

0.89

33%

2

0.80

1.69

45

3

0.71

2.40

15

4

0.64

3.04

7

The expected incremental sales will provide a discounted, net-of-tax contribution margin over 4 years of A. $273,600. B. $92,160. C. $437,760. D. $57,600. A. $273,600 results from subtracting both the incremental fixed cost and the incremental variable cost from the incremental revenue, netting out the tax effect, and discounting the resulting amount as an annuity for four years. However, the $90,000 in incremental fixed cost should not be included in the calculation of a contribution margin. The (c) HOCK international, page 38

Part 2 : Capital Budgeting Methods contribution margin is revenue less variable costs only. B. $92,160 results from subtracting the variable cost of $360,000 from the fourth year's incremental revenue of $600,000. Multiplying the result ($240,000) by (1 − .40) provides a net-of-tax amount of $144,000. $144,000 discounted for 4 years using the present value of $1 factor for 12% for 4 years of .64 equals $92,160. However, the present value of an annuity factor should be used to discount the equal payment stream instead. C. Incremental revenue per year is expected to be 30,000 units × $20 per unit, or $600,000 per year. Incremental variable expense is expected to be 30,000 units × $12 per unit, or $360,000 per year. The incremental (before tax) contribution margin is $600,000 minus $360,000 per year, or $240,000. Income tax is 40%, so the after-tax contribution margin is $240,000 × (1 − .40), or $144,000 per year. Since the amount is the same for each year, the annual amount can be discounted as an annuity. So the discounted, net-of-tax contribution margin over 4 years, using the present value of an annuity factor of 3.04 for four years, is (144,000 × 3.04) = $437,760. D. $57,600 results from subtracting $90,000 and $360,000 from the fourth year's incremental revenue of $600,000. Multiplying the result ($150,000) by (1 ? .40) provides a net-of-tax amount of $90,000, which when discounted for four years using a factor of .64 yields $57,600. However, there are two problems with this approach. First, the $90,000 in incremental fixed costs should not be subtracted from the incremental revenue to calculate the contribution margin, because it is a fixed cost. Second, the question asks for the discounted, net-of-tax contribution margin over four years, not for the fourth year only.

Question 63 - CMA 693 4.25 - Capital Budgeting Methods The following selected data pertain to a 4-year project being considered by Metro Industries: • A depreciable asset that costs $1,200,000 will be acquired on January 1. The asset, which is expected to have a $200,000 salvage value at the end of 4 years, qualifies as 3-year property under the Modified Accelerated Cost Recovery System (MACRS). • The new asset will replace an existing asset that has a tax basis of $150,000 and can be sold on the same January 1 for $180,000. • The project is expected to provide added annual sales of 30,000 units at $20. Additional cash operating costs are: variable, $12 per unit; fixed, $90,000 per year. • A $50,000 working capital investment that is fully recoverable at the end of the fourth year is required. Metro is subject to a 40% income tax rate and rounds all computations to the nearest dollar. Assume that any gain or loss affects the taxes paid at the end of the year in which it occurred. The company uses the net present value method to analyze investments and will employ the following factors and rates.

Period

Present Value of Present Value of $1 at 12% $1 Annuity at 12% MACRS

1

0.89

0.89

33%

2

0.80

1.69

45

3

0.71

2.40

15

4

0.64

3.04

7

The overall discounted-cash-flow impact of the working capital investment on Metro's project is A. $(50,000). B. $(18,000). C. $(59,200). (c) HOCK international, page 39

Part 2 : Capital Budgeting Methods D. $(2,800). A. The cash outflow of $50,000 will occur in Year 0. However, there will be a cash inflow in Year 4 which will reduce the net discounted cash flow amount. B. The Year 0 cash outflow will be $50,000. The $50,000 working capital investment will be recovered at the end of the fourth year. Therefore, the discounted cash inflow will be ($50,000 × .64), or $32,000. So the net overall discounted cash flow impact of the working capital investment will be $32,000 − $50,000, or $(18,000). C. With a cash outflow of $50,000 in Year 0 and a cash inflow in Year 4, the net discounted cash flow for working capital cannot possibly be an outflow that is greater than the initial outflow. D. The net discounted cash flow for working capital needs to include the $50,000 cash outflow in Year 0 net of the present value of the cash inflow in Year 4.

Question 64 - CMA 693 4.27 - Capital Budgeting Methods The payback reciprocal can be used to approximate a project's A. Internal rate of return if the cash flow pattern is relatively stable. B. Net present value. C. Accounting rate of return if the cash flow pattern is relatively stable. D. Profitability index. A. The Payback Reciprocal is 1/Payback Period. (Note: the reciprocal of any number is the number which, when multiplied by the first number, returns a product of 1.) The payback reciprocal can be used to approximate the internal rate of return if (1) the cash flows are equal in every period, and (2) the project's life is at least twice as long as the Payback Period. B. The Net Present Value of a project is the expected monetary gain or loss from a project when all the expected future cash inflows and outflows are discounted to the present point in time, using the required rate of return. Since it is a monetary amount, it cannot be approximated by the Payback Reciprocal, which is a ratio. C. The Accounting Rate of Return is the ratio of the amount of increased book income to the required investment. Thus it is based upon accrual net income, whereas the Payback Method and the Payback Reciprocal are based upon cash flow. D. The Profitability Index is a benefit-cost ratio. It is the ratio of the present value of net future cash flows to the amount of the initial investment. It cannot be approximated by means of the Payback Reciprocal.

Question 65 - CMA 693 4.28 - Capital Budgeting Methods When evaluating projects, breakeven time is best described as A. The point where cumulative cash inflows on a project equal total cash outflows. B. Annual fixed costs ÷ monthly contribution margin. C. The point at which discounted cumulative cash inflows on a project equal discounted total cash outflows. D. Project investment ÷ annual net cash inflows. A. The point where cumulative cash inflows on a project are equal to total cash outflows is the payback period, not the breakeven time.

(c) HOCK international, page 40

Part 2 : Capital Budgeting Methods B. Annual fixed costs ÷ monthly contribution margin is a meaningless calculation. Annual fixed costs ÷ the unit contribution margin results in the breakeven point in number of units to be sold per year to break even. And annual fixed costs ÷ the contribution margin ratio results in the breakeven point in terms of dollars of revenue. However, breakeven time is a completely different concept from breakeven point. C. The breakeven point is also called the discounted payback period. The payback method (undiscounted) determines the number of periods that must pass before the net after-tax cash inflows from the investment will equal (or "pay back") the initial investment cost. However, the payback method has a weakness: it does not consider the time value of money. The breakeven time, or discounted payback method, is an attempt to deal with that weakness. The discounted payback method uses the present value of cash flows instead of cash flows in calculating the payback period. Each year's cash flow is discounted using the required rate of return, and those discounted cash flows are used to calculate the payback period. D. If and only if the annual net cash inflows from a project are all the same, the Project Investment ÷ the Annual Net Cash Inflows equals the Payback Period. However, this is different from breakeven time.

Question 66 - CMA 693 4.29 - Capital Budgeting Methods Flex Corporation is studying a capital acquisition proposal in which newly acquired assets will be depreciated using the straight-line method. Which one of the following statements about the proposal would be incorrect if a switch is made to the Modified Accelerated Cost Recovery System (MACRS)? A. The internal rate of return will increase. B. The profitability index will decrease. C. The payback period will be shortened. D. The net present value will increase. A. The internal rate of return will increase. Since MACRS is an accelerated depreciation method used in calculating income tax liability, its use will result in lower tax liability during the early years of the asset's life than if the depreciation is calculated using the straight-line method. The decreased income tax liability will result in higher cash inflows during the early years of the project. The higher cash inflows during the early years of the project will result in a higher present value of the cash inflows for the project. Since the internal rate of return is the discount rate at which the net present value of the project is zero, the higher present value of the cash inflows for the project will increase the internal rate of return. B. The profitability index will increase. Since MACRS is an accelerated depreciation method used in calculating income tax liability, its use will result in lower tax liability during the early years of the asset's life than if the depreciation is calculated using the straight-line method. The decreased income tax liability will result in higher cash inflows during the early years of the project. The higher cash inflows during the early years of the project will result in a higher present value of the cash inflows for the project. Since the profitability index is the present value of the future net cash flows divided by the initial investment, the higher present value for the cash inflows will result in a higher profitability index. C. The payback period will be shortened. Since MACRS is an accelerated depreciation method used in calculating income tax liability, its use will result in lower tax liability during the early years of the asset's life than if the depreciation is calculated using the straight-line method. The decreased income tax liability will result in higher cash inflows during the early years of the project. The higher cash inflows during the early years of the project will result in a shorter payback period. D. The net present value will increase. Since MACRS is an accelerated depreciation method used in calculating income tax liability, its use will result in lower tax liability during the early years of the asset's life than if the depreciation is calculated using the straight-line method. The decreased income tax liability will result in higher cash inflows during the early years of the project. The higher cash inflows during the early years of the project will result in a higher present value of the cash inflows for the project. The net present value of a project is the expected monetary gain or loss from the project calculated by discounting all expected future cash inflows and outflows to the present point in time, using the (c) HOCK international, page 41

Part 2 : Capital Budgeting Methods required rate of return. Therefore, a higher present value for the cash inflows will result in a higher net present value for the project.

Question 67 - CMA 693 4.30, adapted - Capital Budgeting Methods Jasper Company has a payback goal of 3 years on new equipment acquisitions. A new sorter is being evaluated that costs $450,000 and has a 5-year life. Straight-line depreciation will be used; no salvage is anticipated. Jasper is subject to a 40% income tax rate. To meet the company's payback goal, the sorter must generate reductions in annual cash operating costs of A. $100,000. B. $114,000. C. $190,000. D. $150,000. A. $100,000 in annual before-tax cash flow would equal $60,000 in annual after-tax cash flow. When $60,000 is added to the depreciation tax shield of $36,000 per year, the resulting $96,000 of annual net cash flow increase would not achieve a payback period of 3 years. B. $114,000 is the amount of increased cash flow from operations net of tax that is required. However, it is not the amount that the sorter must generate in annual reductions of cash operating costs in order to achieve a three-year payback. C. The payback period is the number of years it takes for the initial investment in a project to be returned, without considering the time value of money. The new equipment will cost $450,000, and the company needs that amount returned within three years. The depreciation tax shield will be $36,000 per year ($450,000 ÷ 5 × .40). Therefore, in three years, $108,000 of the investment will have been returned by means of the depreciation tax shield, leaving $342,000 to be returned in increased cash flow after taxes. Jasper's tax rate is 40%. Therefore, the before-tax increased cash flow over the three year payback period needs to be $342,000 ÷ (1 − .40), or $570,000. The annual increase in cash flow needed, then, is $570,000 ÷ 3, or $190,000. D. $150,000 is the net after-tax cash flow increase required each year to achieve a payback period of 3 years on a $450,000 investment. However, that is not what the question asks.

Question 68 - CMA 694 4.15 - Capital Budgeting Methods All of the following are the rates used in net present value analysis except for the A. Discount rate. B. Accounting rate of return. C. Hurdle rate. D. Cost of capital. A. The discount rate is used in net present value analysis. B. In net present value analysis, the present value of future cash flows less the net cost of the investment equals the NPV. The rate used to discount the future cash flows to their present value is called the required rate of return. It can also be referred to as the discount rate or the hurdle rate. The cost of capital is frequently used as the required rate of return. The accounting rate of return is not a part of net present value analysis, however. C. The hurdle rate is another name for the required rate of return, or the discount rate, that is used in net present value analysis. (c) HOCK international, page 42

Part 2 : Capital Budgeting Methods

D. The cost of capital is frequently used as the required rate of return in discounting future cash flows to their present value. Thus, it is used in net present value analysis.

Question 69 - CMA 694 4.16 - Capital Budgeting Methods The internal rate of return (IRR) is the A. Rate of interest for which the net present value is greater than 1.0. B. Rate of return generated from the operational cash flows. C. Rate of interest for which the net present value is equal to zero. D. Hurdle rate. A. This is not the definition of the internal rate of return. The internal rate of return is the discount rate that results in a present value of the future cash inflows that is equal to the initial cash outflow. B. This is not the definition of the internal rate of return. Operational cash flows are only one portion of the information used to calculate the internal rate of return. C. The internal rate of return is the discount rate at which the net present value of the project is zero. This means that at that particular discount rate used to discount the future cash inflows to their present value, the present value of the cash inflows equals the initial cash outflow. D. This is not the definition of the internal rate of return. The hurdle rate is another term for the required rate of return used by a firm to discount future cash inflows back to their present value in calculating a project's net present value.

Question 70 - CMA 695 4.1 - Capital Budgeting Methods An advantage of the net present value method over the internal rate of return model in discounted cash flow analysis is that the net present value method A. Can be used when there is no constant rate of return required for each year of the project. B. Uses a discount rate that equates the discounted cash inflows with the outflows. C. Uses discounted cash flows whereas the internal rate of return model does not. D. Computes a desired rate of return for capital projects. A. Since each year's net cash flow is calculated individually and may be discounted individually using net present value analysis, the net present value method can incorporate varying rates of returns during the various years of the project's life. In contrast, the internal rate of return method is used to determine the single discount rate at which the net present value of a project is zero. Thus, the internal rate of return model cannot be used when there is no constant rate of return required for each year of a project. B. The internal rate of return is the discount rate that equates the discounted cash inflows with the outflows for a project. However, the net present value method does not use the project's internal rate of return as the discount rate when determining the present value of the cash inflows and outflows. C. Both the net present value method and the internal rate of return method use discounted cash flows. D. The net present value method does not compute a desired rate of return for capital projects. The desired rate of return must be determined by management and then is used to determine the present value of the cash flows, both positive and negative.

(c) HOCK international, page 43

Part 2 : Capital Budgeting Methods

Question 71 - CMA 695 4.2 - Capital Budgeting Methods Sensitivity analysis, if used with capital projects, A. Is used extensively when cash flows are known with certainty. B. Measures the change in the discounted cash flows when using the discounted payback method rather than the net present value method. C. Is a "what-if" technique that asks how a given outcome will change if the original estimates of the capital budgeting model are changed. D. Is a technique used to rank capital expenditure requests. A. Sensitivity analysis is used when cash flows are uncertain, as a means to determine what a change in one assumption will have on cash flows. B. Sensitivity analysis does not measure the change in the discounted cash flows when using the discounted payback method rather than the net present value method. It can be used with any of the capital budgeting methods to determine how cash flows can be expected to vary with changes in the underlying assumptions. C. Sensitivity analysis can be used in capital budgeting to determine how cash flows can be expected to vary with changes in the underlying assumptions. Sensitivity analysis is a "what if" technique. Using expected cash flows, the NPV, IRR, and PI of the project are determined. Then, the key assumptions that were used in making the original expected cash flow projections are identified. One assumption at a time is then changed, leaving the other assumptions unchanged, and the NPV, IRR and PI are recalculated to determine what effect changing one assumption would have on those measures. D. Sensitivity analysis is not used to rank capital expenditure requests.

Question 72 - CMA 695 4.3 - Capital Budgeting Methods The use of an accelerated method instead of the straight-line method of depreciation in computing the net present value of a project has the effect of A. Increasing the cash outflows at the initial point of the project. B. Lowering the net present value of the project. C. Increasing the present value of the depreciation tax shield. D. Raising the hurdle rate necessary to justify the project. A. Cash outflows at the initial point of the project are not affected by the method used to depreciate it later. B. The use of an accelerated method of depreciation in computing the net present value of a project does not lower the net present value of the project. It has the opposite effect. C. The depreciation tax shield is the amount by which the tax payment owed by the company will be reduced as a result of the tax deductibility of the depreciation. This reduction of the tax payable is essentially a cash inflow for the company. The use of an accelerated method of depreciation for tax purposes will result in higher depreciation in the early years of a project and lower depreciation in the later years, when compared with straight-line depreciation. Higher cash flow in the early years of a project has a greater present value than higher cash flow in the later years. Thus, accelerated depreciation for tax purposes will increase the present value of the depreciation tax shield. D. The use of an accelerated method of depreciation in computing the net present value of a project has no effect on the hurdle rate. The hurdle rate, or the required rate of return, to be used to calculate the net present value of a capital (c) HOCK international, page 44

Part 2 : Capital Budgeting Methods budgeting project is determined by management. It is usually based on the company's cost of capital and is adjusted to reflect the amount of risk perceived by management.

Question 73 - CMA 695 4.4 - Capital Budgeting Methods The profitability index (present value index) A. Is calculated by dividing the discounted profits by the cash outflows. B. Represents the ratio of the discounted net cash outflows to cash inflows. C. Is the relationship between the net discounted cash inflows less the discounted cash outflows divided by the discounted cash outflows. D. Is the ratio of the discounted net cash inflows to discounted cash outflows. A. The profitability index utilizes cash flow information, not profits. B. The Profitability Index is a benefit-cost ratio. The ratio of the discounted net cash outflows to cash inflows would be a cost-benefit ratio. Furthermore, the cash inflows used in the calculation of the Profitability Index are discounted. C. If the discounted cash outflows are in the numerator, they should not also be in the denominator. D. The Profitability Index is a benefit-cost ratio. It is the ratio of the present value of cash inflows to the present value of cash outflows.

Question 74 - CMA 695 4.5 - Capital Budgeting Methods McLean Inc. is considering the purchase of a new machine that will cost $160,000. The machine has an estimated useful life of 3 years. Assume that 30% of the depreciable base will be depreciated in the first year, 40% in the second year, and 30% in the third year. The new machine will have a $10,000 resale value at the end of its estimated useful life. The machine is expected to save the company $85,000 per year in operating expenses. McLean uses a 40% estimated income tax rate and a 16% hurdle rate to evaluate capital projects. Discount rates for a 16% rate are as follows: Present Value of an Present Value of $1 Ordinary Annuity of $1 Year 1

.862

.862

Year 2

.743

1.605

Year 3

.641

2.246

What is the net present value of this project? A. $8,834 B. $30,910 C. $5,842 D. $6,270 A. An answer of $8,834 results from not calculating income tax due on the disposition of the equipment at the end of 3 years. However, there will be a gain on the sale, since 100% of the asset's cost will have been depreciated for tax purposes by the end of the third year.

(c) HOCK international, page 45

Part 2 : Capital Budgeting Methods B. An answer of $30,910 results from including only the initial investment and the gross operating cash flows in the calculation of the net present value. C. An answer of $5,842 results from subtracting the resale value of the equipment from the purchase cost to calculate the depreciable base for tax purposes. For tax purposes, 100% of an asset's cost is always depreciated. D. The cash flows are as follows: Year 0 Initial Investment Depreciation Depreciation Tax Shield (Depr. * .40) Cash from disposition (after tax) Operating cash flows Tax on operating cash flow at 40% Net Cash Flow Discount factor: 16% Discounted Cash Flow

Year 1

Year 2

Year 3

(160,000)

(160,000)

48,000 19,200

64,000 25,600

85,000 (34,000) 70,200 .862 60,512

85,000 (34,000) 76,600 .743 56,914

48,000 19,200 6,000 85,000 (34,000) 76,200 .641 48,844

The net present value is $(160,000) + $60,512 + $56,914 + $48,844 = $6,270

Question 75 - CMA 695 4.6 (adapted) - Capital Budgeting Methods McLean Inc. is considering the purchase of a new machine that will cost $160,000. The machine has an estimated useful life of 3 years. Assume that 30% of the depreciable base will be depreciated in the first year, 40% in the second year, and 30% in the third year. The new machine will have a $10,000 resale value at the end of its estimated useful life. The machine is expected to save the company $85,000 per year in operating expenses. McLean uses a 40% estimated income tax rate and a 16% hurdle rate to evaluate capital projects. Discount rates for a 16% rate are as follows: Present Value of an Present Value of $1 Ordinary Annuity of $1 Year 1

.862

.862

Year 2

.743

1.605

Year 3

.641

2.246

The payback period for this investment would be A. 1.53 years. B. 1.88 years. C. 3.00 years. D. 2.19 years. A. An answer of 1.53 years results from using the annual depreciation instead of the depreciation tax shield as an increase to the annual net cash flows.

(c) HOCK international, page 46

Part 2 : Capital Budgeting Methods B. An answer of 1.88 years results from dividing the net investment of $160,000 by the annual operating cash flow of $85,000. However, the operating cash flow of $85,000 is not the only component of cash flow to be used to calculate the payback period. The annual cash flows are not uniform over the life of this project. C. Since the useful life of the machine is 3 years, an answer of 3.00 would result from using the annual depreciation as the annual cash flow. D. The cash flows are as follows: Year 0

Year 1 Year 2

Year 3

Initial Investment (160,000) Depreciation 48,000 64,000 48,000 Depreciation Tax Shield (Depr. * .40) 19,200 25,600 19,200 Cash from disposition RECD @ YEAR END (after 6,000 tax) Operating cash flows 85,000 85,000 85,000 Tax on operating cash flow at 40% (34,000) (34,000) (34,000) Net Cash Flow excluding cash from disposition (160,000) 70,200 76,600 70,200 Cumulative Cash Flow excluding cash from disposition (undiscounted) (160,000) (89,800) (13,200) 57,000

When calculating the payback period, operating cash flows are usually assumed to be received evenly throughout each year of the project's life. However, the $6,000 received from disposition of the asset is not received until the end of the project, which is at the end of Year 3. Therefore, it is handled differently from operating cash flows. It is not included in the calculation of the payback period in this case, because it occurs at the end of the year, while operating cash flows are assumed to occur evenly throughout the year. Thus, the payback period would end before the disposition occurs. Note that it is not a part of the net cash flow used to calculate the payback period. The cumulative cash flow from the project becomes positive during year 3. The payback period is 2.19 years, calculated as follows: Number of the project year in the final year when cash flow is negative, which is 2 Plus: a fraction consisting of Numerator = the positive value of the negative cumulative inflow amount from the final negative year - 13,200 Denominator = cash flow (excluding disposition) for the following year: 70,200 OR : 2 + (13,200/70,200) = 2.19 Since the disposition of the asset occurs after the payback period, it is not a part of the payback period calculation.

Question 76 - CMA 695 4.7 - Capital Budgeting Methods Capital Invest Inc. uses a 12% hurdle rate for all capital expenditures and has done the following analysis for four projects for the upcoming year. Project 1 Project 2 Project 3 Project 4

(c) HOCK international, page 47

Part 2 : Capital Budgeting Methods Initial capital outlay

$200,000 $298,000 $248,000 $272,000

Annual net cash inflows Year 1

$65,000 $100,000

$80,000

$95,000

Year 2

70,000

135,000

95,000

125,000

Year 3

80,000

90,000

90,000

90,000

Year 4

40,000

65,000

80,000

60,000

Net present value

(3,798)

4,276

14,064

14,662

Profitability Index

98%

101%

106%

105%

Internal rate of return

11%

13%

14%

15%

Which project(s) should Capital Invest Inc. undertake during the upcoming year assuming it has no budget restrictions? A. Projects 1, 3, and 4. B. Projects 2, 3, and 4. C. Projects 1, 2, and 3. D. All of the projects. A. No project with a negative NPV should be accepted, and Project 1 has a negative NPV. B. When a company has no restrictions on its capital investments, it should undertake all projects with positive NPVs, because any project with a positive NPV will increase shareholder wealth. Projects 2, 3 and 4 all have positive NPVs, so all should be accepted. Note also that the Internal Rates of Return for Projects 2, 3 and 4 are all greater than Capital Invest's 12% hurdle rate; and the Profitability Indices for Projects 2, 3 and 4 are greater than 100%. C. No project with a negative NPV should be accepted, and Project 1 has a negative NPV. D. No project with a negative NPV should be accepted, and Project 1 has a negative NPV.

Question 77 - CMA 695 4.8 - Capital Budgeting Methods Capital Invest Inc. uses a 12% hurdle rate for all capital expenditures and has done the following analysis for four projects for the upcoming year. Project 1 Project 2 Project 3 Project 4 Initial capital outlay

$200,000 $298,000 $248,000 $272,000

Annual net cash inflows Year 1

$65,000 $100,000

$80,000

$95,000

Year 2

70,000

135,000

95,000

125,000

Year 3

80,000

90,000

90,000

90,000

Year 4

40,000

65,000

80,000

60,000

Net present value

(3,798)

4,276

14,064

14,662

Profitability Index

98%

101%

106%

105%

Internal rate of return

11%

13%

14%

15%

Which project(s) should Capital Invest Inc. undertake during the upcoming year if it has only $600,000 of funds available? A. Projects 3 and 4. B. Projects 2 and 3. (c) HOCK international, page 48

Part 2 : Capital Budgeting Methods C. Projects 2, 3, and 4. D. Projects 1 and 3. A. When capital is limited, the decision as to which project should receive money first is made using the Profitability Indices of the different proposed projects. (Note that this is not an absolute rule, and there are some exceptions. However, this is not one of the exceptions.) Project 3 and Project 4 will use $520,000 of capital, which is within the $600,000 maximum. Their Profitability Indices are the highest, at 106% and 105%, respectively. In addition, their NPVs and IRRs are the highest. B. This is not the combination of projects that will result in the greatest maximization of shareholder wealth. C. Projects 2, 3 and 4 require more capital than the $600,000 that is available. D. No project with a negative NPV should be accepted, and Project 1 has a negative NPV.

Question 78 - CMA 695 4.9 - Capital Budgeting Methods Capital Invest Inc. uses a 12% hurdle rate for all capital expenditures and has done the following analysis for four projects for the upcoming year. Project 1 Project 2 Project 3 Project 4 Initial capital outlay

$200,000 $298,000 $248,000 $272,000

Annual net cash inflows Year 1

$80,000

$95,000

Year 2

$65,000 $100,000 70,000

135,000

95,000

125,000

Year 3

80,000

90,000

90,000

90,000

Year 4

40,000

65,000

80,000

60,000

Net present value

(3,798)

4,276

14,064

14,662

Profitability Index

98%

101%

106%

105%

Internal rate of return

11%

13%

14%

15%

Which project(s) should Capital Invest Inc. undertake during the upcoming year if it has only $300,000 of capital funds available? A. Project 3. B. Project 1. C. Projects 3 and 4. D. Projects 2, 3, and 4. A. The Profitability Index represents the ratio of the benefits (net cash inflows) to the costs (net initial investment). Thus, it can identify the project with the greatest return per dollar of investment. The Profitability Index enables us to rank different sized investments, since the Profitability Index expresses profitability on a percentage basis rather than a total dollar amount basis. It is very useful for comparing multiple investments that are of different investment amounts. In this case, we can accept only one project because of the limitation in capital. The project with the highest Profitability Index is Project 3, even though it does not have the highest NPV or IRR. Although its NPV is lower than the NPV of Project 4, Project 3 requires only $248,000 of capital outlay compared with $272,000 required for Project 4. B. No project with a negative NPV should be accepted, and Project 1 has a negative NPV. C. Capital Invest has only $300,000 to invest, and Projects 3 and 4 require more capital than the amount available.

(c) HOCK international, page 49

Part 2 : Capital Budgeting Methods D. Capital Invest has only $300,000 to invest, and Projects 2, 3 and 4 require more capital than the amount available.

Question 79 - CMA 696 4.23 - Capital Budgeting Methods Jorelle Company's financial staff has been requested to review a proposed investment in new capital equipment. Applicable financial data is presented below. There will be no salvage value at the end of the investment's life and, due to realistic depreciation practices, it is estimated that the salvage value and net book value are equal at the end of each year. All cash flows are assumed to take place at the end of each year. For investment proposals, Jorelle uses a 12% after-tax target rate of return. Investment Proposal Purchase Cost and Year Book Value

Annual Net After-Tax Cash Flow $

0

Annual Net Income

0

$250,000

$

0

1

168,000

120,000

35,000

2

100,000

108,000

39,000

3

50,000

96,000

43,000

4

18,000

84,000

47,000

5

0

72,000

51,000

Discount Factors for a 12% Rate of Return: Present Value of Present Value of Annuity of $1 $1 Received at Received at End Year the End of Period of Each Period 1

.89

.89

2

.80

1.69

3

.71

2.40

4

.64

3.04

5

.57

3.61

6

.51

4.12

The accounting rate of return on the average investment proposal is A. 17.2% B. 12.0% C. 28.0% D. 34.4% A. An answer of 17.2% results from using the initial investment in the denominator of the calculation of the accounting rate of return on investment. This is one way of calculating the accounting rate of return. However, in this problem, we are told to use the average investment amount, not the initial investment amount. B. 12% is Jorelle's after-tax target rate of return. C. An answer of 28% results from dividing the minimum increase in the expected annual after-tax accounting net income ($35,000) by the average investment of $125,000.

(c) HOCK international, page 50

Part 2 : Capital Budgeting Methods D. The accounting rate of return on an investment, using the average investment amount as instructed in the question, is the increase in the expected annual average after-tax accounting net income divided by the average investment. Since the initial investment is $250,000, the average investment is $250,000 / 2, or $125,000. The average after-tax accounting net income attributable to the investment is ($35,000 + $39,000 + $43,000 + $47,000 + $51,000) / 5, or $43,000. $43,000 / $125,000 = .344 or 34.4%.

Question 80 - CMA 696 4.24 - Capital Budgeting Methods Jorelle Company's financial staff has been requested to review a proposed investment in new capital equipment. Applicable financial data is presented below. There will be no salvage value at the end of the investment's life and, due to realistic depreciation practices, it is estimated that the salvage value and net book value are equal at the end of each year. All cash flows are assumed to take place at the end of each year. For investment proposals, Jorelle uses a 12% after-tax target rate of return. Investment Proposal Purchase Cost and Year Book Value

Annual Net After-Tax Cash Flows $

0

Annual Net Income

0

$250,000

1

168,000

120,000

$

35,000

0

2

100,000

108,000

39,000

3

50,000

96,000

43,000

4

18,000

84,000

47,000

5

0

72,000

51,000

Discounted Factors for a 12% Rate of Return: Present Value of Present Value of Annuity of $1 $1 Received at Received at End Year the End of Period of Each Period 1

.89

.89

2

.80

1.69

3

.71

2.40

4

.64

3.04

5

.57

3.61

6

.51

4.12

The net present value for the investment proposal is A. $(97,970) B. $106,160 C. $96,560 D. $356,160 A. An NPV of $(97,970) results from using accounting net income rather than cash flows in the analysis. B. The NPV is the present value of the future cash inflows less the initial investment, when the future cash flows are discounted using the discount rate determined by management. In this case, the discount rate chosen by management is 12%. Using the present value of $1 factors, the present value of the future cash flows is ($120,000 × .89) + ($108,000 × .80) + ($96,000 × .71) + ($84,000 × .64) + ($72,000 ×.57) = $356,160. Subtracting the (c) HOCK international, page 51

Part 2 : Capital Budgeting Methods initial investment of $250,000 from the present value of the future cash flows of $356,160 gives us an NPV of $106,160. C. An answer of $96,560 results from using the average of the annual after-tax cash flows and multiplying it by the factor for an annuity discounted at 12% for 5 years, then subtracting the initial investment amount. However, in calculating net present value, each individual year's cash flow must be discounted separately. D. $356,160 is the present value of the future cash flows, but it is not the net present value for the investment proposal.

Question 81 - CMA 696 4.25 - Capital Budgeting Methods Jorelle Company's financial staff has been requested to review a proposed investment in new capital equipment. Applicable financial data is presented below. There will be no salvage value at the end of the investment's life and, due to realistic depreciation practices, it is estimated that the salvage value and net book value are equal at the end of each year. All cash flows are assumed to take place at the end of each year. For investment proposals, Jorelle uses a 12% after-tax target rate of return. Investment Proposal Year

Purchase Cost and Annual Net Annual Flows Book Value After-Tax Cash Net Income

0

$250,000

$

0

$

0

1

168,000

120,000

2

100,000

108,000

39,000

3

50,000

96,000

43,000

4

18,000

84,000

47,000

5

0

72,000

51,000

35,000

Discounted Factors for a 12% Rate of Return: Present Value of Present Value of Annuity of $1 $1 Received at Received at End Year the End of Period of Each Period 1

.89

.89

2

.80

1.69

3

.71

2.40

4

.64

3.04

5

.57

3.61

6

.51

4.12

The traditional payback period for the investment proposal is A. Over 5 years. B. 2.83 years. C. 2.23 years. D. 1.65 years. A. An answer of "over 5 years" could result from using the annual net income amounts instead of the annual after-tax cash flows to calculate the payback period. B. An answer of 2.83 years results from using the discounted annual after-tax cash flows to determine the payback (c) HOCK international, page 52

Part 2 : Capital Budgeting Methods period. However, the traditional payback period uses undiscounted cash flows to calculate the payback period. C. When cash flows are not constant over the life of the project, we must add up the cash inflows in order to determine when the inflows will equal the outflows. The cash flows are as follows: Year 0 Year 1 Year 2 Year 3 Year 4 Year 5 Net initial investment (250,000) After-tax cash flows from operations 120,000 108,000 96,000 84,000 72,000 Cumulative cash flows (250,000) (130,000) (22,000) 74,000 158,000 230,000 The cumulative cash flow from the project becomes positive sometime during year 3. If the cash flows are assumed to occur evenly throughout the year, the exact payback period is 2.23 years, calculated as follows: Number of the project year in the final year when cash flow is negative: 2 Plus: a fraction consisting of Numerator = the positive value of the negative cumulative inflow amount from the final negative year: 22,000 Denominator = cash flow for the following year: 96,000 OR: 2 + (22,000/96,000) = 2.23 The initial investment will be recouped after 2.23 years. D. An answer of 1.65 years results from adding together the annual after-tax cash flow and the annual net income for each year and using the total to determine the payback period. However, the payback period should be determined using annual after-tax cash flow only.

Question 82 - CMA 696 4.26 - Capital Budgeting Methods Which one of the following capital investment evaluation methods does not take the time value of money into consideration? A. Internal rate of return. B. Discounted payback. C. Net present value. D. Accounting rate of return. A. The internal rate of return is the discount rate at which the net present value of an investment project is zero. Therefore, it does take into consideration the time value of money. B. The discounted payback method does take into consideration the time value of money, because it is calculated using discounted cash flows. C. Net present value does take into consideration the time value of money, because it equals the present value of future cash inflows minus the net initial investment. D. The accounting rate of return is the average annual increased accounting net income after tax attributable to the investment divided by the net initial investment, or sometimes divided by the average investment over the life of the project (i.e., the initial investment amount divided by 2). The accounting rate of return does not take (c) HOCK international, page 53

Part 2 : Capital Budgeting Methods into consideration the time value of money.

Question 83 - CMA Sample Q4.4 - Capital Budgeting Methods Jackson Corporation uses net present value techniques in evaluating its capital investment projects. The company is considering a new equipment acquisition that will cost $100,000, fully installed, and have a zero salvage value at the end of its five-year productive life. Jackson will depreciate the equipment on a straight-line basis for both financial and tax purposes. Jackson estimates $70,000 in annual recurring operating cash income and $20,000 in annual recurring operating cash expenses. Jackson's cost of capital is 12% and its effective income tax rate is 40%. What is the net present value of this investment on an after-tax basis? A. $80,250 B. $8,150 C. $36,990 D. $28,840 A. This answer uses only the before-tax operating cash flow to calculate the net present value. This ignores both the depreciation tax shield and tax on the operating cash flow. B. $8,150 ignores the depreciation tax savings. C. Annual after-tax cash flows will be [($70,000 − $20,000) × (1 − .40)], which equals $30,000. The annual depreciation tax shield is ($100,000 / 5) × .40, which is $8,000. Therefore, the total annual after-tax cash flow is $38,000. The present value of these net inflows for a 5-year period is $136,990 ($38,000 x 3.605 present value of an ordinary annuity for 5 years at 12%), and the NPV of the investment is $36,990 ($136,990 − $100,000 initial investment). D. $28,840 is the present value of the depreciation tax savings.

Question 84 - CMA Sample Q4.5 - Capital Budgeting Methods Mega Inc., a large conglomerate with operating divisions in many industries, uses risk-adjusted discount rates in evaluating capital investment decisions. Consider the following statements concerning Mega's use of risk-adjusted discount rates. I. Mega may accept some investments with internal rates of return less than Mega's overall average cost of capital. II. Discount rates vary depending on the type of investment. III. Mega may reject some investments with internal rates of return greater than the cost of capital. IV. Discount rates may vary depending on the division. Which of the above statements are correct? A. II and IV only. B. I and III only. C. I, II, III, and IV. D. II, III, and IV only.

(c) HOCK international, page 54

Part 2 : Capital Budgeting Methods A. The company may accept some projects with IRRs less than the cost of capital, or reject some project with IRRs greater than the cost of capital. B. Risk-adjusted rates may also reflect the differing degrees of risk, not only among investments, but by the same investments undertaken by different organizational subunits. C. All of the statements are correct. When we analyze risk, we are measuring the amount of variability that could take place in the future returns from the investment being considered. One way of accounting for the anticipated amount of risk in a capital budgeting analysis is to use a required rate of return to discount the cash flows that is either higher or lower than the company's cost of capital. If the project is considered to be riskier than the firm's existing business (there is more variability in the possible future cash flows), a higher required rate of return is used to discount the future cash flows. If the project is considered to be less risky than the firm's existing business, a lower required rate of return is used to discount the future cash flows. This is called a "risk-adjusted rate of return." Mega may accept some investments with internal rates of return less than Mega's overall average cost of capital because the investment may be considered to be less risky than the firm's other business, and thus a lower required rate of return (i.e., hurdle rate) was used to discount its expected future cash flows. Discount rates vary depending on the type of investment because a higher discount rate would be used to discount a riskier investment and a lower discount rate would be used to discount a less risky investment. Mega may reject some investments with internal rates of return greater than the cost of capital because those investments were judged to be riskier than the firm's existing business and therefore the expected future cash flows have been discounted at a rate higher than the firm's cost of capital. For a riskier investment using a risk-adjusted discount rate, the expected cash flow from the investment will need to be relatively larger, or the increased discount rate will generate a negative net present value. So even though the investments' IRRs are greater than the cost of capital, if they are not greater than the higher required rate of return, the investments will be rejected. Discount rates may vary depending on the division because the same investments undertaken by different organizational subunits may carry different levels of risk due to risk factors that could be specific to one unit and not to another unit. D. The company may accept some projects with IRRs less than its cost of capital.

Question 85 - IMA 08.P3.170 - Capital Budgeting Methods The following methods are used to evaluate capital investment projects. • • • •

Internal rate of return (IRR) Average rate of return (ARR) Payback Net present value (NPV)

Which one of the following correctly identifies the methods that utilize discounted cash-flow (DCF) techniques? A. IRR and NPV. B. IRR and ARR only. C. IRR and Payback only. D. Payback and NPV only.

(c) HOCK international, page 55

Part 2 : Capital Budgeting Methods A. Both the IRR and NPV use discounted cash-flow techniques. B. ARR does not use discounted cash-flow techniques. C. Payback does not use discounted cash-flow techniques. D. Payback does not use discounted cash-flow techniques.

Question 86 - IMA 08.P3.171 - Capital Budgeting Methods Which one of the following methods for evaluating capital projects is the least useful from an investment analysis point of view? A. Net present value. B. Payback. C. Internal rate of return. D. Accounting rate of return. A. The net present value method uses cash flows in its analysis and is therefore a useful method for investment analysis. B. The payback method uses cash flows in its analysis and is therefore a useful method for investment analysis. C. The internal rate of return method uses cash flows in its analysis and is therefore a useful method for investment analysis. D. Accounting rate of return uses accounting profit in its analysis instead of cash flows. This makes this the least useful of the methods listed for investment analysis.

Question 87 - IMA 08.P3.176.01 176 - Capital Budgeting Methods Olson Industries needs to add a small plant to accommodate a special contract to supply building materials over a five-year period. The required initial cash outlays at Time 0 are as follows: Land $ 500,000 New building 2,000,000 Equipment 3,000,000 Olson uses straight-line depreciation for tax purposes and will depreciate the building over 10 years and the equipment over 5 years. Olson's effective tax rate is 40%. Revenues from the special contract are estimated at $1.2 million annually, and cash expenses are estimated at $300,000 annually. At the end of the fifth year, the assumed sales values of the land and building are $800,000 and $500,000, respectively. It is further assumed the equipment will be removed at a cost of $50,000 and sold for $300,000. As Olson utilizes the net present value (NPV) method to analyze investments, the net cash flow for period 3 would be A. $940,000 B. $880,000 C. $860,000 D. $680,000 A. This answer results from depreciating the building over a 5 year period instead of over 10 years. (c) HOCK international, page 56

Part 2 : Capital Budgeting Methods B. This answer results from depreciating the land along with the building. However, land is never depreciated, either for tax purposes or book purposes. C. The third year's cash flow would be: Cash flow from operations after tax: ($1,200,000 − $300,000) × .6 = $540,000 Depreciation tax shield: Depreciation on building = $2,000,000 ÷ 10 = $200,000 per year for 10 years1 Depreciation on equipment = $3,000,000 ÷ 5 = $600,000 per year for 5 years1 Total depreciation in Year 3 would be $200,000 + $600,000, or $800,0002 The depreciation tax shield in Year 3 is $800,000 × .4, or $320,000 Total cash flow for Year 3 is $540,000 cash flow from operations after tax + $320,000 depreciation tax shield, or $860,000. 1

For tax purposes, the depreciable base is 100% of an asset's cost, regardless of which depreciation method is used and regardless of whether salvage value is expected.

2

Depreciation on land is not included because land is not depreciated.

D. This answer results from multiplying the pre-tax net cash flow by the tax rate, .40, to calculate the after-tax net cash flow. The pre-tax cash flow should be multiplied by 1 − the tax rate to calculate after-tax cash flow.

Question 88 - IMA 08.P3.176.02 194 - Capital Budgeting Methods Olson Industries needs to add a small plant to accommodate a special contract to supply building materials over a five-year period. The required initial cash outlays at Time 0 are as follows: Land $ 500,000 New building 2,000,000 Equipment 3,000,000 Olson uses straight-line depreciation for tax purposes and will depreciate the building over 10 years and the equipment over 5 years. Olson's effective tax rate is 40%. Revenues from the special contract are estimated at $1.2 million annually, and cash expenses are estimated at $300,000 annually. At the end of the fifth year, the assumed sales values of the land and building are $800,000 and $500,000, respectively. It is further assumed the equipment will be removed at a cost of $50,000 and sold for $300,000. As Olson utilizes the net present value (NPV) method to analyze investments, the net cash flow for period 5 would be A. $2,070,000 B. $2,230,000 C. $2,390,000 D. $1,710,000 A. This answer results from depreciating the building over a 5 year period instead of over 10 years. B. This answer is incorrect. See correct answer for an explanation. C. (c) HOCK international, page 57

Part 2 : Capital Budgeting Methods The fifth year's net cash flow would be: Cash flow from operations after tax: ($1,200,000 − $300,000) × .6 = $540,000 Depreciation tax shield: Depreciation on building = $2,000,000 ÷ 10 = $200,000 per year for 10 years1 Depreciation on equipment = $3,000,000 ÷ 5 = $600,000 per year for 5 years1 Total depreciation in Year 5 would be $200,000 + $600,000, or $800,0002 The depreciation tax shield in Year 5 is $800,000 × .4, or $320,000 Sale of land, net of tax: The land is sold for $800,000 and cost $500,000. Since land is not depreciated, its tax basis is $500,000 (an asset's book value for tax purposes is called its tax basis). There will be a $300,000 gain on the sale that will be taxable at the rate of 40%, which is $120,000. Thus, the net cash flow from the sale of the land will be $800,000 − $120,000, or $680,000. Sale of building, net of tax: The building is sold for $500,000. Its tax basis at the end of the 5th year is $200,000 − (5 × $200,000) = $1,000,000. Since it is sold for $500,000, there is a loss of $500,000 on the sale. The tax benefit from the loss is $500,000 × .40, or $200,000. The net cash flow from the sale of the building is $500,000 + $200,000 tax benefit from the loss, or $700,000. Sale of equipment, net of tax: The equipment will be fully depreciated at the end of the 5th year. It will be sold for a net amount of $250,000 ($300,000 proceeds from sale minus removal cost of $50,000). The full net amount is taxable, so tax on the gain is $250,000 × .4, or $100,000. The net cash flow from the sale will therefore be $250,000 − $100,000, or $150,000. Total cash flow for Year 5 is $540,000 cash flow from operations after tax + $320,000 depreciation tax shield + $680,000 sale of land + $700,000 sale of building + $150,000 sale of equipment = $2,390,000. 1

For tax purposes, the depreciable base is 100% of an asset's cost, regardless of which depreciation method is used and regardless of whether salvage value is expected.

2

Depreciation on land is not included because land is not depreciated.

D. This answer is incorrect. See correct answer for an explanation.

Question 89 - IMA 08.P3.199 - Capital Budgeting Methods The owner of Woofie's Video Rental cannot decide how to project the real costs of opening a rental store in a new shopping mall. The owner knows the capital investment required but is not sure of the returns from a store in a new mall. Historically, the video rental industry has had an inflation rate equal to the economic norm. The owner requires a real internal rate of return of 10%. Inflation is expected to be 3% during the next few years. The industry expects a new store to show a growth rate, without inflation, of 8%. First year revenues at the new store are expected to be $400,000. The revenues for the second year, using both the real rate approach and the nominal rate approach, respectively, would be A. $432,000 real and $444,960 nominal. B. $440,000 real and $452,000 nominal. C. $440,000 real and $453,200 nominal. D. $432,000 real and $452,000 nominal. A. The information that the owner requires a real internal rate of return of 10% is irrelevant to solving this problem. The relevant rates are the 8% real growth rate expected in the industry and the expected 3% inflation

(c) HOCK international, page 58

Part 2 : Capital Budgeting Methods rate. The Year 2 real revenue will be the Year 1 revenue multiplied by 1 + the expected industry growth rate of 8%: $400,000 × 1.08, or $432,000. The Year 2 nominal revenue will be the Year 2 real revenue multiplied by 1 + the expected inflation rate: $432,000 × 1.03, or $444,960. B. The Year 2 real revenue is calculated as the Year 1 revenue multiplied by 1 + the owner’s required real internal rate of return of 10%, which is not correct. The owner’s required real internal rate of return is not relevant for this analysis. The forecasted industry growth rate of 8% should be used. The Year 2 nominal revenue of $452,000 is calculated as $400,000 × (1 + .10 + .13), which is incorrect. The nominal revenue for Year 2 should be calculated as the real revenue for Year 2 multiplied by 1 + the inflation rate. C. The Year 2 real revenue is calculated as the Year 1 revenue multiplied by 1 + the owner’s required real internal rate of return of 10%, which is not correct. The owner’s required real internal rate of return is not relevant for this analysis. The forecasted industry growth rate of 8% should be used. The Year 2 nominal revenue of $453,200 is calculated as the incorrect Year 2 real revenue of $440,000 × 1 + the inflation rate. The correct Year 2 real revenue should be multiplied by 1 + the inflation rate to calculate the correct Year 2 nominal revenue. D. The nominal revenue for Year 2 is calculated as $400,000 × (1 + .10 + .13), which is incorrect. The nominal revenue for Year 2 should be calculated as the real revenue for Year 2 multiplied by 1 + the inflation rate.

Question 90 - IMA 08.P3.202 - Capital Budgeting Methods The net present value profiles of projects A and B are as follows. Discount Rate Net Present Value $(000) Project A Project B (percent) 0 $2,220 $1,240 10 681 507 12 495 411 14 335 327 16 197 252 18 77 186 20 (26) 128 22 (115) 76 24 (193) 30 26 (260) (11) 28 (318) (47) The approximate internal rates of return for Projects A and B, respectively, are A. 19.0% and 21.5% B. 20.5% and 26.5% C. 19.5% and 25.5% D. 0% and 0% A. At a discount rate of 19.0%, Project A has a Net Present Value of approximately 25.5. At a discount rate of 21.5%, (c) HOCK international, page 59

Part 2 : Capital Budgeting Methods Project B has a Net Present Value of approximately 89. These NPVs were calculated by interpolating from the data given. The internal rate of return is the discount rate at which the NPV is equal to zero; so 19.0% cannot be the internal rate of return for Project A and 21.5% cannot be the internal rate of return for Project B. Look for the rates where the NPV of each project changes from a positive number to a negative number. The rate at which the NPV is zero will be somewhere in between those two rates. B. At a discount rate of 20.50%, Project A has a Net Present Value of approximately (48.25). At a discount rate of 26.5%, Project B has a Net Present Value of approximately (20). These NPVs were calculated by interpolating from the data given. The internal rate of return is the discount rate at which the NPV is equal to zero; so 20.5% cannot be the internal rate of return for Project A and 26.5% cannot be the internal rate of return for Project B. Look for the rates where the NPV of each project changes from a positive number to a negative number. The rate at which the NPV is zero will be somewhere in between those two rates. C. The internal rate of return is the discount rate at which the NPV is equal to zero. We are given several NPVs for both projects, each one calculated using a different discount rate. As the discount rate increases, the NPVs decrease. For each project, the rate at which the NPV changes from a positive number to a negative number will be approximately where the NPV will be zero. The IRR will be a rate in between the last positive rate in the table and the first negative rate. We can estimate that from the table. For Project A, at a discount rate of 18%, its NPV is 77. At a discount rate of 20%, its NPV is (26). Therefore, the discount rate at which its NPV will be zero must be somewhere in between 18% and 20%. For Project B, at a discount rate of 24%, its NPV is 30. At a discount rate of 26%, its NPV is (11). Therefore, the discount rate at which its NPV will be zero must be somewhere in between 24% and 26%. The only answer choice that fulfills both of those ranges is 19.5% and 25.5%. With the answer choices given, there is no need to do any further calculation to interpolate the exact IRR for each project. D. At a discount rate of 0% for both projects, Project A has a Net Present Value of $2,220 and Project B has a Net Present Value of $1,240. The internal rate of return is the discount rate at which the NPV is equal to zero; so 0% cannot be the internal rate of return for either project. Look for the rates where the NPV of each project changes from a positive number to a negative number. The rate at which the NPV is zero will be somewhere in between those two rates.

Question 91 - IMA 08.P3.215 - Capital Budgeting Methods Fred Kratz just completed a capital investment analysis for the acquisition of new material handling equipment. The equipment is expected to cost $1,000,000 and be used for eight years. Kratz reviewed the net present value (NPV) analysis with Bill Dolan, Vice President of Finance. The analysis shows that the tax shield for this investment has a positive NPV of $200,000, using the firm's hurdle rate of 20%. Dolan noticed that 8 year straight-line depreciation was used for tax purposes but, since this equipment qualifies for 3-year MACRS treatment, the tax shield analysis should be revised. The company has an effective tax rate of 40%. The MACRS rates for 3-year property are as follows. Year Rate 1 33.33% 2 44.45% 3 14.81% 4 7.41%

Accordingly, the revised NPV for the tax shield (rounded to the nearest thousand) should be A. $109,000 (c) HOCK international, page 60

Part 2 : Capital Budgeting Methods B. $192,000 C. $283,000 D. $425,000 A. This answer is incorrect. See correct answer for an explanation. B. This answer is incorrect. See correct answer for an explanation. C. The tax shield is calculated as follows: Year 2 Year 3 Year 4 Total Year 1 MACRS Depreciation 33.33% 44.45% 14.81% 7.41% Depreciation $333,300 $444,500 $148,100 $74,100 × .40 = Depreciation tax shield $133,320 $177,800 $59,240 $29,640 × Discount Factor .833 .694 .579 .482 PV of Depreciation Tax Shield $111,056 $123,393 $34,300 $14,286 $283,035

D. This answer is incorrect. See correct answer for an explanation.

Question 92 - IMA 08.P3.224 - Capital Budgeting Methods Parker Industries is analyzing a $200,000 equipment investment to produce a new product for the next 5 years. A study of expected annual after tax cash flows from the project produced the following data. Annual After-Tax Cash Flow Probability -----------------------------$45,000 .10 $50,000 .20 $55,000 .30 $60,000 .20 $65,000 .10 $70,000 .10 If Parker utilizes a 14% hurdle rate, the probability of achieving a positive net present value is A. 30% B. 60% C. 20% D. 40% A. This is the probability that the annual cash flow will be $55,000. However, an annual cash flow of $55,000 will not result in a positive NPV for the project. B. This is the probability that the annual cash flow will be $55,000 or below. An annual cash flow of $55,000 or below will not result in a positive NPV for the project. C. This is the probability that the annual cash flow will be $60,000. An annual cash flow of $60,000 will result in a positive NPV. However, other annual cash flows will also result in positive NPVs, and the answer to the question should be the (c) HOCK international, page 61

Part 2 : Capital Budgeting Methods total of all annual cash flows that will result in positive NPVs. D. This question involves six different possible 5-year annuities – the six probabilities. We need to find which of them will result in positive NPVs and then sum the probabilities of those cash flows to find the total probability of having a positive NPV. It is not necessary to calculate an NPV for each possible cash flow in order to do that, however. A positive NPV would be a present value of the positive cash flows that is greater than the initial expenditure of $200,000. We need to find the lowest annual cash flow that results in a positive NPV. That annual cash flow and any cash flow above it will result in a positive NPV, and the total of those cash flow probabilities will give us the total probability of the NPV being positive. To find the annual cash flow that results in a positive NPV, we can divide the $200,000 original investment by the factor for a 5-year annuity at a discount rate of 14%, which is 3.433. The result is $58,258. Therefore, any annual after-tax cash flow that is greater than $58,258 will produce a positive net present value. Annual after-tax cash flows of $60,000, $65,000 and $70,000 all are above $58,258. Therefore, the probability of achieving a positive net present value is the sum of the probabilities of these three annual after-tax cash flows: .20 + .10 + .10, which equals .40 or 40%.

(c) HOCK international, page 62

View more...

Comments

Copyright ©2017 KUPDF Inc.
SUPPORT KUPDF